You are on page 1of 130
CHAPTER 1 THE POWER SYSTEM: AN OVERVIEW 1.1 INTRODUCTION ‘States was established in 1882 atthe Peart Street Station in New York City by Thomas Edison, The station supplied de po ting the lower Manhattan area. The power was generated by de 2. 1 THROWER SYSTEM: ANOVERVIEW ‘The first single-phase ac system in the United States was at Oregon City where power was generated by two 300 np watervnees turbines and trans at 4 KV to Portland. Souther Califorma Edison Company mstalled the phase system at 2.3 KV in 1893, Many electric companies were developed trotgt- ‘out the county, In the beginning, individual compemes were operating at di throoghout the U.S.and Canada. Me tem, Transmission voitages have (EHV) in commercial use 18 765 KV, 1969. For transmitting power over very long distances t may be more economical to ‘convert the EHV ac to EHV cc, transmit the power over two lines, and invert tac at ine other end. Studies show that 1s advantegeous to consider de between the two ngid systems ac links, The main disedvantag requires filterng, and a large amount of reactive power compensation required &t both ends ofthe line. The first 400-KV dbo line mn the United States was tne Pacitic Inter, 850 miles tong between Oregon and Califorma built n 1970. ‘The entire continental called tne power grid. A smal ‘owned, but the bulk 1s privately owned. The system aphical regons called power pools. In an aterconnect {ors are required as a reserve for peak load ana spinning reserve. lion makes the energy generation and transmission more ezonor be cheaper for a company to buy bulk power from neighboring ullities than 10 ‘promice 11m one ofits older plants. 1.2. ELECTRIC INDUSTRY STRUCTURE “The bulk generation of electricity in tne Unites States 1s produced by untegrated investor-owned ules (TOU). A small portion of power generation 1s federally ‘ownea, such as the Tennessee Valley Authority and Bonnevilie Power Admnistra- tion, Two separate levels of regulation currently regulate the United States electric system: One 1s the Federal’ Energy Regulatory Commission (FERC), which reg- Fecent progress in techn Res and opportunt- 41. THE POWER SYSTEM: AN OVERVIEW ‘aes for power systems research and open up new opportunities to young power engineers. 1.3 MODERN POWER SYSTEM ‘The power system of ogay is a complex interconnectee network as snown in Figure 1.1 (Page 7). A power system canbe subivided into four major pats: sion and Subtransmission '* Disuibuuon + Loads 13.1 GENERATION Generators — One of the essential components of power systems 18 the theee= hase ac generator known as syncnronous generator or alternator. Synchronous generators have two synchronously rotating fields: One field 1s produced by the rotor driven at synchronous speed and excited by de current The other file is pro- duced in the stator windings by the three-phase armator curtents, The de current 1 provided by excitation systems. In the ol ‘6c generators mounted on the same shaft, providing rushless excatation systems, The generator excita voltage and controls the reactive power flow. Because they lack the commutaror, tc generators can generate high power at high voltage, typically 30 kV. Ina power plant, te size of generators can vary from 50 MW to 1500 MW. ‘The source of the mechanical power, commonly known as the prme mover may be hydraulic turbines at waterfalls, steam turbines whose fe turbines, particularly With a low pressure, operate at ow speed. Their generators are usually as 'ype rotor with many poles. In a power station several generators aze operated parallel in the power grid to provide the total power needed: They are connected st ‘common pornt called a bus 12 MODERN POWER SYSTEM. 5 the average power of a 2815 W is equivalent co energy sources of alternate sources which are neing used to Hiydro and pumped si Gas Torbine possible 61. THE POWER SYSTEM: AN OVERVIEW ‘The insulation requirements and other practical design pro generates s, usually 30 kV. Thus, step-up sed for transmission of power. At tne receiving en ofthe transi down transformers are used to reduce the voltage to suitable value ‘or utilization. In a mogem utility system, the power may undergo, formations petween generator ana ultimate user 1.32. TRANSMISSION AND SUBTRANSMISSION ‘The purpose of an ovethead transmission network 15 10 from generating units at vanous locations to the distribu sfer electne energy 9 ‘mately supplies the load, Transmussion lines also inerconnett neighboring ut \whien permits not only economic dispatcn of power within regrons dunn, conditions, bat also the transfer of power Fignre 1.1 shows an elementary diagram of a transmission anc di referred to as switching stations. A down to value more suitable forthe subtransmission voltage levels. Typically, the subtransmassion vo from 69 to 138 kV, Some targe inaustral customers may ve 1.33 DISTRIBUTION ‘The distribution system 1s mat part which connects the di the consumers’ servce-entrance equipment. The pnimary di ally in tne range of 4 to 34.5 kV and supply the load in aw area. Some small inaustiat ‘The secondary commercial and resident 1 MODERN POWER SYSTEM 7 (cat 69-138 kV BIGURE f.1 Basie comrenets a pow system, 8 | THE POWER SYSTEM: AN OVERVIEW red feet m length then deliver power to the individual consumers. The secondary distribution serves most of the customers at levels af 240/120 V, three-wire; 208Y/120 V, three-phase, four-wire; or 480¥/277 Y, th atypical home 1s denved from a transformer primary feeaer vottage 10 240/120 V using a tnree-wire line Distribution systems are both overhead and underground. The grow erground distribution has been extremely rapid and as much a5 70 pe residential construction is served underground. 134 LOADS Loads of power systems are divided into maustial, commercial, and resideatal, ‘ery large maustnal Loads industrial toads are serves di Joads are composite loads, and induction motors form a high proportion of foaa. These composite inads are functions of voltage und frequency and fo ‘mayor part of the system load, Commercial and residential Joads consist largely of lighting, neaung, and cooling. These toads ate mdepeadent of freaucney and ‘The magnitude of load vanes throughout the day, and power must be avail ‘consumers on demand, ‘of maximum demand. Smaller peaking generators may be cor tne peak load that occurs for only a few hours. In order to as ‘of the generating plant the foad factor 18 defined. The load factor average load over a designated period of tume fo the peak foad occuring an that period. Load factors may be gwven for a day, a month, or @ year. The yearty, or ‘annual load factor 1s the raost useful since a year represents a full cycle of time. ‘The daily load factor 1s peak ‘Multiptying the numerator and denominator of nave toaa Daily LE. = an ) by a time period of 24 hr. we average load x 24hr _ energy consumed dung 24 hr Dally LF. = “Peak load % 24 hr peak load 5.24 hr i ‘The annual load factor is “Annuat LF. = tat anual cuergy an peak toad » 8760 br 5 MoobRN eowsn svsrem 9 Generally theres diversity in \whion imp the rato of (0 of annual te how well ‘non obtains a piot of the ‘me load must be defined 2 Use the barevele function to obtain a pl we g0ven data compute the average load and the n- Ram, ‘The following commana ata=[0 2 6 2B 12016 10. 4, THEFOWER SYSTEM: AN OVERVIEW 4 16 14 16 18 16 18 20 18 20 22 16 2223 12 23 2 6) s P= aata(:,3); % Colum array of loa 1:43); % Golam array of cemana .aterval w= Pespe; % Total energy, area uncer the curve Pavg = W/eun(Dt) Average 1020 eens aneaes ts wl ul é | ae | 7 | (ca | 4g 5 10 15 20 25 nouns Daily oad cyte for Exam Pavg = 11.5417 Peak = 18 LF = 64.12 L4,SYSTEM PROTECTION 1 14 SYSTEM PROTECTION her devices at system. Some ¢ a computer. Simul are tnplemented fa ‘an electrical system, Flow nodel all components of etec- ving the concepts of power oust trans- the perfor \d generators are a part of ‘of @ power system, sls operation is methods of power systema se of MATLAB, The MAT: sm mathematical expression 124, THE FOWSR SYSTEM: AN OVERVIEW {o sumutation, Some of tne basic analysis covered in this text are «© Evaluation of transmission line parameters «+ Transmission line performance and compensation «+ Power flow analysis «+ Beonomic scheduling of generation + Synenronous machine ranstent analysis ‘+ Baanced fautt «+ Symmetrical components and unbalanced fanit + Stability studies + Power system contr Many MATLAB functions are developed forthe above tne student to concentrate on analysis and design of practic less time on programming, PROBLEMS JA. The demand estimation 1s the starung potnt for planning the future power supply, The consistency of demand gro to numerous attempts to fit matnematical curves samplest curves is P= Po ners is tho average per unt growth rate, Piste demand mn year ae isthe given demand at year to. “Assume the peak power dermand inte Unt States 1984 480 an average growth rate of 3.4 percent. Using MATLAB, pot me peak demand in GW from 1984 to 1999, Estimate he peck pow for ihe yéar 1999, 1.2. Ina certain country, the energy consumption 15 expected to double 1 ‘yeats. Assuming a simple exponential grov'tn given by Pa Pot wet calculate the growth rate a. @ S.COMPUTER ANALYSIS 1 le. During eat ye, Using MATLA es 2 BASIC PRINCIPLES 21 INTRODUCTION “The concept of power is of eenal the main topc of this chapter, The tpic of this material, anc the review nere encountered inthe electric circuit th Tn this enaptr, the flow of energy in an ac cicurt 1s mvestigatea. By using various trigonometne identities the mstantanecus power p(@) 1s resolved in ‘component. A plot ofthese components s obtaned using ac networks nol only consume energy at an average rate, ‘energy (0s souroes. This leads to the basie definitions of average power reacuie power Q. The vott-ampere S, which is a mathematical formulation based sncal power systems. already have studied inforee me power balance 1s demonstrated, and the transmit ow power factors are discussed and demonstrated by means of severat examples. Next, the transmission of complex power between evo voltage sources 1s com: sidered, and the dependency of real power on tne vol pendency of reactive power on voltage magnitude 1s es conveniently to demonstrate this idea graphic Finally, the balanced three-phase ci of a balanced three-phase system 1s tha s constant power. That 1s, the WT INSINGLE-FHASE ACCIRCUNTS If power delivered dos not fluctuate purpose of analysis anc mods the three-phase system under balanced consition single-phase system, For tn suits developed fe 2.2 POWER IN SINGLE-PHASE AC CIRCUITS Figure 2.1 shows & single-pnase si ate) FIGURE 21 s are Supping le Let the mstantaneous voitage be vt) = Qu lang the snstantaneous cucrent be given by 1 ys co 22 We prodvet of vattage v(t) 3a) ing the tngonometrc identity 03 Aas B= j ons ~ 8) + con + B) en 16 2. BasiceRicirLEs which results m 1 Dit) = 5 Veal |ces(By — 6) + cos(2ut + By + &)]} = Nites 8) cst +) (O4~89) = HWallas(y ~ 63) + os (ut +8} cslBy ~ 8) sin ut + 6,)sin(8 — smean-square (rms) vai 0 = Vou ana the rms vale of = Im/ V2. Let @ = (0, — 9;). The above equation, in terms of the rms ss reduced 0 9{t) = [VI [Neos 6 + cos Aut + 6)] + [Villain an Aut + & Pat) ptt) a Energy flow ino Energy borrowed ang the exreurt rotumed by the cxreuit, where 0 1s the angle between voltage and current, or the impedance angle. @ 1s if the load is capaci ‘The instantaneous power nas been decomposed into two components. The first component of (2.5) alt) = VI loos6 + [V7]. c0s #008 2( 26) “The second tem mn (2.6), whic hs equeney twice a of he source. accounts 2 fore nnaoidal vanation inthe abssepion of owe by te resisive poon cf the toad, Since the average value of this sinusoidal function 1s zero, ine average power delivered to the load is given by Pa |Vlillcos 0 an ‘This is the power absorbed by tre resistive component of the load and 15 a ferred to as the active power of real power. The product of the rms voltage val ‘and the rms current value [V|)7| is called the apparent power and is measured ‘units of volt ampere, The product ofthe apparent power and the cosine of the anal ‘between voltage and current yields the real power. Because cos @ plays a key rale power factor. When the current red lagging. When the current leas the “The secona component of x(t) = A careful study of Equatc Isucs of the mstantaneots power tne voltage by 90°, and the of energy from el = 100cosct ana the load 1s mine the expression for the Use MATLAB to plot 18 2 BASICPRINCILES (6) = Voy cosut, it) = Im cos{ut — 60) 100 om 30 4000] o 2000 50 ° 100 “e300 400-200 ot degree pr(t), Eq. 26 fs Py oe 0 1002008004100 BD) KTaO et, degree FIGURE22 tnsantaneous erent, voltage power, Eas. 2.6 and 28. therefore and 4 Voltage amplitude ana pnase angle 4 Impedance magnitude ana phase angle Corrent pnase angle an cogres % Degree to radian % Current anplituae Yovt fron 0 to 2xp2 Y Tastentancous voltage 25,COMPLEXFOWER | (ut + tnetarsps/180) stantaneous curxen~: astantaneous powes joltage and curren ‘Average powe 1 Reactive pove ‘¢ Coupiex powe %Eg, (2.6 ieq. 8 power of lengen w for ple: x generates a zero vecto: ¥ grad 2.3 COMPLEX POWER ‘The rms voltage paasor of (2.1) and the 2.2) shown i Fi ure 2.3 are 7 V= 1120, anc ee a aa FIGURE23 Prasoc dagram an 20 2, Basic PRINCIPLES = [VIL] cos @ + IV [|Z] sn 6 “The above equation defines a complex quantity where 1 real parts the average (teal) power P and its imaginary partis the reactive power (). Thus, the compen power designated by Sis gwen by S=VvI=P+jQ ‘The magnitude of S, |S] = VP2F G2, 1s the apparent power; its unt amperes and the larger units are KVA or MVA. Apparent power gives a «ation of heating and is used as a rauing unit of power equipment. has practical significance for an electnic ntility company since @ and apparent power to consumers current impedance and J lags V). Q is negate when @ is negative (L2., when the load impedance 15 capacitive and J Teacs V) as shown in Figure 24 In working with Equation (2.10 } its convement to think of P, Q. and 5 as forrming the sides of a nght tangle as shown in Figures 2.3 and 24. r v AX “ Q 5 aia If the load impedance 15 Z then var ny 1 for V into (2.10) yields S=VP =ZIr = Rup +5XiTh ‘evident that complex power $ and impedance Z have tne same angle, Because the power tnangle and the ampedance tangle are similar tangles, the impedance angie is sometimes called the power angle. ‘Similarty, supstnating for I from (2.11) mto (2.10) yrelds| vv _ sevreeatt O13) SA THE COMPLEX POWER BALANCE 21 From (2.13), te impedance of te comoiex power $is given by ue Thus the total compiex power © Complex powers deliverea to 15) = 120020" fe ai 4D A 22 2 RASICPRINCIFLES Sy = VE} = 120020°(20 — j0) = 24,000 W+ j0 var Sp = VIZ = 1200£0°(40 + 780) = 48, 000 W + 496, 000 var Sq = VF = 1200.£0°(20 ~ j20) = 24, 000 W ~ 324,000 var ‘The total load complex power adds up to S = 5, +5;+ Se = 98,000 W 772,000 var ‘the sum of complex power delivered to the load ean be obtained by otal current Ts I+ Lr Jy = (20+ 50) + (40 ~ 780) + (20-4 920} = 80 — 760 = 1002-36.87" A 5 = VI" = (120020°)(100296.87°) = 120,00036.87° VA = 96,000 W + 72, DOO var ‘A final insight is contained in Figure 2.6, which shows the current phasor diagram and the complex power vector representation, S FIGURE 26 (Curent sor diagram and power pane diagra The compiex powers may aso be obtained diet from (2.14) Se we ar = 24,000 W470 S= we = eee = 48,000 W + 96,000 var s- Me or = 24,000.W ~ 524,000 var 5 TOWER FACTOR CORRECTION. 2 2.5 POWER FACTOR CORRECTION will be larger f power P suppl thou addition: custome as close 10 1 5 power compante ‘They alse impose & power factors, Sine power Factor at source, and the W450 var = 800 W 4 92600 var PRINCIPLES 5 24 2. Baste 2.5 POWBICFACTOR CORRECTION 25 ‘Total apparent power and current are 2 S = P+ JQ = 1200 + 71600 = 2000/53.13° VA © 5 2000-8213 45) a3 490 le 1 F< ROT 100-5818" A | Power factor tte source is i PE = cos(53.13) = 0.6 lagging | 2. (0) Find re copecstanceof the capnertor connected across ie loa to unprove the overall power factor 10 0.8 agg. | “ota real power P = 1200 W at ine new power factor 0.8 lagging, Therefore Ui 6! = cos*(0.8) = 36.87" Q = P tan! = 1200+an(36.87*) = 900 var ' For Load i Qe = 1600 ~ 900 = 700 var I = cos | ‘The toad comalex powers are Wh 9120 evar ‘The total power and the new current are ‘S' = 1200 + {900 = 1500296.87" i S=P+IQ= 54 = (85+ [Note the reduction in the supply curtent from 10 A t07.5 A. Example 2.4 ‘Three loads ate connected 1m parallel across a 1400-V rms, 60-Elz smngie-phase supply as shown in Figure 2.8. Load 1: Inductive 1034, 125 kVA at 0.28 power factor. Load 2: Ca oad, 10 KW and 40 kvas Load 3: Resistive foad of 15 kW. (@) Find the total kW, kvar, KVA, anal the supply power factor. 26 2. BASIC PRINCES ‘otal reat power P = 60 KW atthe new power factor of 0.8 lagging results im the new reactive power Q! ‘Therefore, the required capacitor kvar 18 Qe= 80-45 = 35 Kvar = 53.571 -36.87" A [Note the reduction in the supply current from 71.43 A to $3.57 A. 2.6 COMPLEX POWER FLOW Consider two ideal voltage sources connected by a line of impedance Z = B+ 4X as shown in Figure 2.9. Z=RejX=lZi oe OQ» FIGURE 29 “Two uetconaeted voltage sures. Let the phasor voltage be Vi = |i] 6, and V2 = |V2|déz. For the assumed direc- mn of current 26 COMPLEX POWER FLOW ‘The compiex power Sue is given by Sas I= ‘Thus, ne real and re Pe osly +51 ~ da) ene Qe ~&2) er Power system tran Assuming R= 2.18) i) 219) Since = 0, there are no trensmission ne real power recerved, From the above res Ing important opser 808 and the real power sent equals stem with small R/-X rato, the 4, Equation (2.18) snows that smal effect on the real power flow, ‘not have appreciable effect on th Power on trnsmmsston 1 ‘ne terminal voltage 5 is positive anc ne real power flows fram noce 's negative anc power flows from node 2 10 node 4 Assuming Re 0, the theorete power transfer is given by (220) a 6 statte transmission eapac- wo machines, 28 2 nASIC PRINCWLES 3, For mamtaining transient stability, the power system 1s usually operates with ‘small load angie 6. Also, from (2.19) the by the magnitude difference of terminal voltages, (6. Q 0: [| Example 2.5 ‘Two vottage sources Vj = 120/~5 V and Va = 10020 V are connected by a short line of impedance Z = 1+ 471 as shown in Figure 2.9. Determine the rea) and reactive power suppliee or tecetved by each source and the power loss 1n te line __ 1u0e-5° ~ 10040" In= = 8.195/-110.02" A = 3.195269.98" A = Vilfg = 970.22105.02° = ~97.5 W + 5068.3 var = Vali = 313.5/~69.98" = 107.3 W ~ 32945 var Line 1oss is given oy Sp = 5, +S, =9.8 W+ 5638 vor From the above results, since P; is negative and Pp 15 postive, souree | receives 97.5 W, and source 2 generates 107.3 W and the real power loss in the Tine ts 9.8 'W. The real power loss an the line can pe checke 2, = Rial? = (1)(3.135)? = 9.8 W 1s 363.3 var and source Also, since Qu 8 positive ana Qa seg 1568.6 var The eacve 2receives 294.5 power loss inthe ine can be checked by Qu = Xihal? = (7)(3.135)" = 68.8 var Example 2.6 ‘This example concems the direction of power flow between 10 Write a MATLAB program for the system of Example 2.5 such th gle of source i 1s changed from it ‘magnitudes of the two sources and the voltage plisse angle of source 21s vo be Kept constant. Compute the complex power for each source and the line oss. Tabs the real power and plot F , P:, and P;, versus voltage phase angle ¢, The following: commands 26 COMPLEX POWER FLOW % Lame impeaane by #/- 30, col. arra onvert degree to radia x = bengtn(ad of same lengen for a iver degree co radia qa = (i= Be vate . Bote = amas ‘SL = Siss2, S Bal Besuitl = fai, P eet a9 Eereteae, 258 5 text(-26, 100, 4 1 Vortage tag Source # | Phage togie = Source #2 Voleage fog Source # 2 Pause inpis dette pei po -2800G0 -272"2049 oa 2 = “o-0aco “760-8061 naaore “Becco “eep.e125 Gans “erence “si2.s301 beers “Sse “sve dee2 26 3508 “foleace “a4o.cez8 15 03 ‘Siooce “Soy ston 9 a2es ° ‘ecn00 2.0000 30 2, Basic PRINCIPLES §.0000 195.9349 -185.5084 9.8265 40.0000 243.3751 ~328,0828 18.2928 15.0000 490.9938 ~466.6382 24. 3556 20.0000 637.0676 ~600.1201 36.9475 25.0000 780.4848 -727.5125 62.9728 a a a a ‘Source #1 Voltage Phase Angle Real powe: versus vtige phase angle 6 Examination of Figure 2.10 shows that the flow of real power along the intercon ‘ection is dteriined by the ange difference ofthe terminal voltages. Problem 2.9 requires the development ofa similar program for demonstrating the dependency of reactive power on the magnitude difference of terminal voltages, 2.7 BALANCED THREE-PHASE CIRCUITS “The generation, transmission and distribution of electnc power 1s accor means of three-phase ciremts. At the generating station, three sinusoidal fre generated having the same amplitude but displaced in phase by 120° called a balanced source. Ifthe generated voltages reach thetr peak values sequential order ABC, the generator 1s said t0 have a positive phase sequence, snow in Figure 2.11(@), Ifthe phase order 18 ACB, the generator 1s said to have a negative phase sequence, as shown in Figure 2.11(b). INCED THREE PHASECIRCUTS ly meludes both A onnested, because if the voltage: are vot perfectly by and consequent Consequently a cireutat sng current, around sBcnerato, ana thu generator suppl mnectes 21 wads of transmission Fines of the generator Vin, Vin ‘oad terminals are balanced, (2.22) (2.23) 322 RASICPRINCIPLES FIGURE 2.12 [A Yeconnected generator suppying a Y-coonected Toad 2.8 Y-CONNECTED LOADS ‘To find the relationship between the line vottages (line-to-line voltages} and the phase voltages (line-to-neutral voitages), we assume a positive, or ABC, sequence. We arbitrarily choose the line-to-neutral voltage of the a-phase as tne reference, ius Von o Vin Yon (224) yh —240" represents the magnitude of the phase voltage tages atthe load terminals sn forms ofthe phase vo! of Kirchhof's vortage faw = Ving = [Wol( 120" ~ 14-1209) = V3|¥91/30° 14~120° — 12~240°} = v3|¥p|<—90" (2.25) 1d 240° ~ 120°) = V3}%5) 2150" ‘The vowtage phasor diagram of tne Y-connectea toads of Figure 2.12 1s shown 1m Figure 2.13. The relationship between the line voltages and phase voltages 1s ‘aemonstratea graphically. SALY.CONNECTEDLOADS 2 y Vz, then one of thc Where @ is the impedance phase angle. ‘Phe current hase impedances}. Thu 34 2 Baste PRINCIPLES 2.9 &-CONNECTED LOADS ‘A balanced A-connected load (with equal phase impedances) ure 214, qt, FIGURE 214 [A Acconneted load, Itis clear from the inspection of the cirout thatthe Hine woltages are the same as phase voltages. w=% 2.29) Consider the phasor diagram shown in Figure 2.15, where the phase current arbitrarily chosen as reference. we have on = [ILL Tue = Vold~120° 230) Jog = Mg d-24" where |p] represents the magmitude of the phase current FIGURE 215 Paso diagram showing phase anne cues | | 1 TRANSFORMATION ‘The relationship between phase and Kirchhof?’s current law at the = Vly] £90" ase Currents 1s cemonstra. bby J, ten one of the imps 362, nasiCreMciPLes Seo” FIGURE2I7 ator diagram showing phase and lie vonage, “The phasor diagram in Figure 2.17 shows the relationship between balanced phase and fine-to-line voltages, From this pnasor diagram, we find Van + Vac = VI lVan|£30° + V3 Van é~30° (2.34) = Ven (235) et “Substituting m (2.33), we get _ on - Ga ta on Za on = Ele 3 Now, forthe Y-connected enreut, we have Von = Zvle ean "Thus, from (2.36) and (2.37), we find that ay = 28 238) 2.11 PER-PHASE ANALYSIS “The current inthe neutral of the balanced Y-connected loads shown in Figure 2.12 is given by Ihahthth=0 (239) CED THREES POWER pedance ts + power system problems 2 ther solved on other two phases car identical a single-subscrine 2.12 BALANCED THREE-PHASE PO! Consider a load with the (2.40) an 38 2 BASIC PRINCIPLES where [Vp] and |Z] are the magnitades of the rms phase voltage and curent, re~ spectively. The foal instantaneous power isthe sum of the mstantaneous power of ‘each phase, given by Pap = Vane + Vint + Uente ‘Substituting forthe instantaneous voltages and currents from (2.40) and (2.41) mo 2.42) ‘The three double frequency cosine terms in (2.43) are out of phase with each other bby 120° and add up to zero, and tne unee-pase instantaneous power 1S Pag = SVM ons ean 8 = 0, —8is the angle between phase voltage and phase current or the impedence angle ‘Note that although the power in each phase ss pulsstn ‘neous power is constant and equal fo three times the teat pow ‘deed, this constant power isthe main advantage ofthe three-phase sestexn 0 single-pnase system. Since tne power m each phase is pu 15 made up of the real power and the reactive power. In ord symmetry between real and reactive powers, the concept of complex or epparent power (5) is extended to three-phase systems by defining tne thee-pnase reacte power as I anstanta- Qag = 3WMpll sn 2.45) “Thus, the complex three-phase power s Sag = Pg + 1036 (2.46) Sag = B53 ean Equations (2.48) and (2.45) are sometimes expressed an terms of the rns rmagmitude of the line voltage andthe rms magnitude of the tine curent. n= Y> connected load the phase voltage {V] = [Vc|/V3 and the phase current Jy = I, 212 BALANCED THREE PHASE POWER ing for the phase vo reactive powers for eith aa -4) and (2.45) shows ame for either a quantities. real end reactive powe A thtee-phase line has an ampeéance of 2 2+ 449 [Wi] = 207.85 ‘are connected 1n parallel, The wn from the supply. 40 2 easic PRINCIPLES (©) The line vottage at the combined loads. (e) The current per phase in each load. (@ The toral reat and reactive powers n each load and the line, (2) The A-connected load is transformed into an equivalent Y. The impedance per ‘hase of the equivalent ¥ is 60 = SoS 229-15. 0 ‘The phase vottage 1s ‘The single-phase equivalent cic 1254 tn Ha a 2 Vi = 12020°V we” . n IGURB220 Single-phase ecusaiet ‘The total impecance 1s (30 + j40)(20 — 15) (30 + ja) + GO 715) S24 j4+2- j= UO Bas jae ‘Wit me phase voltage Va a reference, the current phase a8 Yi _ owe 2 ae “The tnree-pase power suplid is 5 = BVI" = 3(12020°)(5.0") = 1800 W T= (b) The phase voltage at tne toad terminal Vy = 12020" — (2+ 44)(SL0") = 110 = 520 = 1118-103" V ANCED THREE PHASE POWER The line vottage Foe = W + j600var 00 W ~ 3900 var TThe mnree-phase power absorbed by So WR 5 50 W + 5300 var equal tothe power delivere #900) + (150-4 5300) 42 > nasicrmincieces O44 5270 vy v= 220040" |5y Sr FIGURE221 ‘Single-phase euaen” agra for Exumpte 2.8 (a) The phase vortage atthe toad terminals 1s 3810.5 y= SIDS = aon v ae ‘The single-phase equivalent circutis showin in Figure 2.21, “The total complex power ts Saige) = 960.1(0.707 + 30-707) +182 = 528 + 3398 = 660088.87° VA \With tne pase voltage Vs as reference, the current nthe fine 1 1 Suu, $80, 0004~ 96:87" aVy 31220020") “The phase voltage at ie sending end is Vj = 2200208 + (0.4 + j2.7)100L—36.87° = 2401.724.58° V = 100/-26.87" A ‘The magnitae ofthe ine voltage atthe sending end oF ne nes Waal = Valv| = VaCed01.7) = A200 Vv (b) The three-phase power oss inte fine 1s Sugagy = SRL + j8X UP? = 8(0.4)(100)? + 9302.7 = 12 KW 4 781 oar () The three-pnase sending power 13 Saas) = BVI" = 9(2401.74,58°)(1003687°) = SAD KW + 3477 kar is clear that the sum of loza powers and te line losses 15 equal (© the power ivered from the supply, 1.¢. Ssis0) = Sassy + St1ap) = (628 + 1396) + (12+ J81) = 540 KW JST? Kya PROBLEMS 2d. Modify the program in Example 2.1 suk be entered by the user ‘The pe: ak am 16 Vg and the 64). The imp that the following. quanaes ean ile By of the sinusoidal supply acnivude Z, and the phase angle (4) and pee), sim- = 100 V.@, = O and the lo ithe ims value of the eurrent sup 44 2 BASIC PRINCIPLES 24, An inductive load consisting of R and X in parallel feeding from a 2400-V ‘ms supply absorns 288 KW at a laggmg power factor of 0.8, Determine R ana X. 2.5, Two loads connected in parallel are supplied from a single-phase 240-V rms source, The two loads draw 2 total real power of 400 KW at a power factor of 0.8 lagging. One of the loads araws 120 kW at a power factor of 0.96 reading, Find the complex power ofthe other load. shown im Figure 2.22 consists of a ee Rin peal of reactance X. The load is fed from a single-phase supp! '2000° V ems, andthe load is taking 30 kVA at 0.8 power factor Fina the vaives of F and X. (b) Determine te supply voltage V 844 sen T : > ooo VER 23x FIGURE 2.22 (Greu for Problem 26, 247. Twoimpedances, Z, = 0.84 5.6 1 and Zp = 8-16 0. and a single-phase ‘motor are connected in parallel across @ 200-V rms, 60-Hz supply as snown 1n Figure 2.23. The motor draws SKVA at 0.8 power facto vt Th hh bs Sy = 5KVA 2ooce* Vv a0.8 PF lag j56} 910 = T FIGURE223 Cec for Pobiem 27. LANCED THREE.PHASE POWER hw oo impedances, and Sy fort the Supply current, a ‘ted by a line of impedane 26° V and V3 nd determine wheth, fe power. Also, find the re, rple 2.6 such enat the vott- tant. Compute the complex the reactive powers and 1m the results, show nis determined by rastantaneous phase vot 46 2, axsic PRINCIPLES supplies a patanced Y-connected load of impedance Z = 250496.87° 0 per phase. (a) Using MATLAB, plot the instantaneous powers Pe Poy Pe a4 versus w# over a range of 0:0.05: 2 on the same graph. nature of the mstantaneous power in cach phase and the total three-phase real power (©) Use (2.44) to verify the total power obtained in pat ( 2A. A 4157-V mms, three-phase supply 1s appliad to a balanced Y-connected three-phase loaa consisting of three identical impedances of 48/36.87°2. ‘Taking the phase to neutral vottage Van, a8 reference, caiculate (@ The phasor currents in each line, (b) The total active and reactive power supplied fo the 1oaa 2.12. Repeat Problem 2.11 with the same three-phase ampedances arranged in a & ‘connection. Take Vay as reference. 243, A balanced delta connected load of 15 + j18 9 per phase 1s connected at the end of a three-phase line as shown in Figure 2.25. The line umpeaance 1s 1 + 52 2 per phase. The line 1s supplied from a three-phase source with a Jine voltage of 207.85 V rms. Taking Vay, as reference. determine te (©) Magnrtude of the lin 1+ 720 Wel = 207.85 FIGURE 225 Cire for Problem 2.13, 2.14, Thtee parallel three-phase loads are supplied from a 207.85-V rms, 60-Hz, tee-phase supply. The loads are as follows: Load 1: A.15 hp motor operating at 0.6 lagging power factor. ad, 93.25 percent efficiency, and 215 BALANCED THREE-HIASE ROWER 47 oral of 6 KW. ating of 36 kvar (a) What isthe cote systent KW, k per phase? ¥. and the supply curren’ ceurrent per pnase wher the capacitor bank 1s lee 2.16 CHAPTER 5 GENERATOR AND TRANSFORMER MODELS; THE PER-UNIT SYSTEM 3.1 INTRODUCTION Before the power systems network can be solved, 1t must frst be modsied, The three-phase balanced system 1s represented on a per-phase Da h was de. seribed in Section 2.10. The single-phase representation 1s also used for unbelancet systems by means of symmetrical components which is treated in 2 iater chapter, In this chapter we deat with the balanced system, where transmission lines are eP- resented dy the » model as described in Chapter 4, Other essentiah components of a power system are generators and transformers; their theory and construction ae discussed in standard electne machine textbooks. In this chapter, we re sample models of generators an transformers for steady-state bala Next we review the one-line diagram of a power system stowan transformers, transmission lines, capacitors, usual stall loads are not shown im detail out are taken into {0ads on substation busses, 48 32, SYNCHRONOUS GENERATORS 49 In the analysis of power to use the per. mn of transformers by 's presented, followed by the unpedance mon MVA base. 3.2, SYNCHRONOUS GENERATORS mous generators, known 2s ‘oF gas turbines. The tor. The armawure phase voltages and are ar- (es as the fleld winging that 1s power (0,23 percent of aiso equipped sly 70 percent of Irom about 150 to fon the poles ang g2 synchronous (500 MVva, ions are driven by 1 The sta- 1 by 120 elec. be considered to ‘50 2, GBNERATOR AND TRANSFORMER MODELS: THE PER-UNCT SYSTEM FIGURE, Elemensary two-cole tree hase synchronous generator, an air gap flux of ¢ per pole and is revolving at constant angular veloc flux linkage of the coil vanes with the position of the rotor minf axis for an atu = 7/2. Assuming distributes cosine of the angle wt. Thus, the flux linkage with coi n= Ne cost ‘The vottage induced in coil aa! is cbtamed from Faraday’s taw as ee =~ OB =u Noanut = Eos st 62 = Emus costwt — 5) where Emax = UNG = In fNO ‘Therefore, the ems value of t a4 ices three-phase is. The frequency of the rotor runs and requency of the B85) During norma) S2_ 5, GENERATOR AND TRANSFORMER MODELS, THE PER-UNTT SYSTEM instantaneous value ofthe phase current, ie, Fy = Kig = Klos sin (ut — U) = Fa sin{ut -V) Fy = Kiy = KIqrsin(ot ~~) = y= Kie = K Imac sin(t — 8 — 4 = Fy sin(ut - where Kis proportional tothe number of armature tums per pase ands fun of the winding type. The resuitant armature mmf is the vector sum of the above rims, A saitable method for finding the resultant mmf to project these mts. on Tine mn and obtan te resultant in-phase and quadrane-prase components. The resultant n-pnase components are 2s, v-F) Bn wv Fy = Fasin(ut~ W)costut—¥) + Fa sin(ut = ¥- ) an te 6) t Fmantet 9 peastut- 0 - Using the trigonometric identity sin a-cos.a = (1/2}sin Ze, the above expression becomes Qt — Y) + sin Yut —w- p pan gut-o © “The above expression 1s the sum of three sinusoidal funcuons dispiaced trom enc couner by 2x /3 radians, which adds up to 2210, 1e., Fi = 0 The sam of quadrature components resus in 2 Fy = Fr sinlut — v)sin(ut ~ ¥) + Fa siolut —¥ ~ Bpaint +Fnsintut -v- Zjsntut—y- 4 3 ‘Using the tngonometne iden: becomes sin? a = (1/2) = c0¢2a), the above expression = Fan eat —v) +oontur 6-8) A F008 2wt ~ v 3 ‘The sinusoidal terms of the above expression are displaced from each other bw 2n/3 radians and add up to zero, with F, = 3/2F. Thus, the amplitude of the resultant armature mint or stator mmf becomes 3 Re=5Fn 38) stant amplitude Synctronisin 1 magnetic fel, type a generator ts helds, When the haces she generaced emf f Ene, known as the Tz by 90° and thus can be represented by a voltage drop across a reactance Xer due to the current Ig. Xar 15 known as the reactance of the armature reaction. The phasor sum of E and Ey, is shown by Eyr perpendicular to Fp, which represents the on-ioad generated emt. B= Ey +jXelo 39) The terminat voltage V is less than By by the amount of resistive voltage drop ‘fe and leakage reactance voltage arop Xf. Thus EaVtlReti(Xet Xerdlfe G10) E=V+|[RotjXs\lo BID where X, = (X¢+ Xer) is known as the synchronous reactance. The cosine angle between 1 and V, 1.e., e088 represents the power factor at the gere ‘minals. The angie between £ and By, 1$ equal tothe angie between tne F, and the aur gap mmf Fy, shown by 6. The power developed by « 15 proportional fo the product of F, Fyr and sin6,..The relative positions of minfs dictates the an angle 6,, the machine 1s operating as a generator and when F. al the machine will act as a motor. Since Hand By, are proporuoral to F, aed Fy the power developed by the machine is proportional to the poate and sin. The angle 6, is thus known as the power angle. This isa very result because at relates the time angle between the phasor emis witb ty of the mac reactance 1s not constant, The unsaturated synchronous react ‘age point on the open: value known as the saturated {equivalent cxreuit connected fo an infinite bus becomes that shown in and (3.11) reduces t0 BaV+GXsIe ey ih terminal vottage cading pf load BIGURE 35 Svachronous gensrat pi ‘a (esminal voltage \d current ted load at some 8.3) ‘S63, GENERATOR AND TRANSFORMER MODELS: THE PER-UNIT SYSTEM the no-load voltage, Since this is not a practical method for very large machines, fan accurate analytical method recommended by IEFE as given in reference [43] may be used. An approximate method that provides reasonable rest sider a hypothetical linearized magnetization curve drawn to intersect the actual ‘Magnetization curve at rated voltage, The value of B calculated from (3.12) then Used to fing the field current from the linearized curve. Finally, the n0-oad voitage corresponding to this field current is found from the actu (0 con: 3.3 STEADY-STATE CHARACTERISTICS— CYLINDRICAL ROTOR 3.3.1 POWER FACTOR CONTROL Most synchronous machines are connected to ‘arge interconnected networks. These networks have the important characteris at the point of connection 1s constant in magnitude, pl Such a poi at the generator bus will not be altered by changes condition, Machine operates as a generat shown in Figure 3, Pag = REVI] = 3] where Vis the piase-to-neutral terminal the «p of the armature current pas ited by varying the field cure a vertical line as the Figure 3.6, From 41 = Xylay cost G15) ‘Thus £; sin y is a constant, and the locus of Ey ts on tne line ef. In Figure 4.6 phasor diagrams are drawn for three armature currents. A\ lagging power factor armature current Ta, results in Ey operates at umity power factor ang arm’ by Ina, which res Si leading power factor. Figure 3.6 shows that the gené "S-- CYLINDRICAL ROTOR 57 a constant real 58 2. GENERATOR AND TRANSFORMER MODELS; THE PER-UNIT SYSTEM ‘Thus, the real power Psp and reactive power Qsg are G19) sin(y = 4) = 6.2m If Re is neglected, then Z, = j X, and y = 90°. Equations (3.19) and (3.20) reduce to Pag = G21 ag = all \E|cos 6 —|V')) 3.23) Equation (3.21) shows that if || and |V| are held fixed and tie power angle 9 is changed by varying the mechanical diving torque, the power lransfer vanes sinusoidally with the angle 5. From (3.21), the theoretteal maximum power occurs when 5 = 90° 82) “The behavior of the synchronous machine ean be ceserbed as follows. If we with 6 = O° and increase tne driving torque, the machine accelerates, an ihe mint F, advances ind, causing reaches equilibrium where the electne power output chanical power owing to the increased driving torque. ‘were made to advance & further than 90° by increasing the diving corque, elecinc power output would decrease from the Pyez Dott. Thee dinving torque conunues to accelerate the machine, andthe ma smageetically coupled. The machine loses synchronism an automatic equipment isconneets t from the system, The value Pras 18 eae tne steady sey dit synchronous machine achie ‘erably less than 90°. The conttol of real power flow is maintained by governor through the requency-power Equation (3.22) shows that for small 4, cos 5 is neatly unt and the reactive power can be approximated to (3.24) the overcxcite reactive power for inductve tea the flow of reactive power ove difference ia won voltage || and the bas by voitage |¥/|.The adjastm power is achieved by the genera Example 3.4 A.50-MVA, 30-KV, theee-ohase, 60. reactance of 9 9 par phase and a rated power st 29.8 power factor bus. (a) Determine the ¢ generator has a synchroncu 2. The generator is deliverin ‘The rated volt (b) When the generator 1s delivering 25 MW from (3.21) the power angie is (25)(9} S= sia" | roa 56 )17 a) = 10.591" ‘The armature current 1s b= (BROS 1 Soe = 807.485/-53.49° The power factor is given by cos(53.43} = 0.596 lagging, (©) The maximum power occurs at 5 == 90° Paste) = SLE = PBS 5 saw cure current 1, = PP: S58690" = 17.5200") sors sscancan™ a 3 “The power fator 1 given by 5T feading Pxample 3.2 ‘The generator of Example 3.1 1s aetivenng 40 MW aca terminal vol Compute the power angle, armature current, and power factor whe ‘ent is adjusted for the following excitations. (a) The excitation votage 1s decreased 0 79.2 percent of the value ple 3.1 (b) The exertation voltage is decreased to 59.27 percent of the vs ample 3.1 (€} Find the minimum exestation below wich the generator will Ise sv (a) The new ex: CYLINDRICAL ROTOR 61 LOA ‘The power fa (b) The new exerts From (3.211 the pot The = 0.37 leading The ge Example 3.3 ig 1egtOM Is given by jown in Figure 3.7, 62. 3, GENERATOR AND TRANSFORMER MODELS; THE PER-Ut P= 40; % real power, MW V = 20/aqee(a)+ 3*0; % pnase voltage, kV 2s = 3#95 synchronous impedance ang = acos(0.4); ‘tnataraag: 0.4 leading to 0.4 legging pf P's Prones(t,lengen(theta)) ifgenerazes array of sate size Tam = P./(3*abs(V)*cos(tneta)) ; % current magmatude kA Ta = Iam.«(cos(theta) + jesin(theta)); —% current pnasor Be V+ Zs.Ta; y exeatation voleage paascr Ea = avs (2); excitation voltage aagastuce, KV IE = £9¥1000/20005 % field current, A plot(Tf, Tam), gric, xlavel(*Tf - a) ylabel Cia - sek), text(3.4, 1, ‘Leading pf pext(13, i, ‘Lagging pf’), text(®, .71, ’Upt?? BIGURE31 V curve for generator of Example 3.2 3.4 SALIEN'T-POLE SYNCHRONOUS GENERATORS “The model developed an Section 3.2 1s only valid for cylindrical recor ge with uniform air gaps. The salient-pote rotor results in nonuniformity nneuc reluctance of the air gap. The reluctance along the polar referred to as the rotor direct aut, is appreciably tess than that along the mterpolar =N'-POLE SYNCHRONOUS GENERATORS 63 Therefore, the reactance ha: mature and can be taken inte Pvo components Ig, in phase, ‘The phasor diagram with the 10 longer possible to rep- account by reso and fa armature resistaee negle a ha a Ne ee L te FIGURE SS Phas disgram tor ieot-role generat Fesent the machine by a simpie equi uo voutage magnitude 6.25) 3.26) n1 be expressed! in terms of Lg and or 64. >, GENERATOR AND TRANSPORMER MODELS: THE FER-UNIT SYSTEM Also, from (3.25), [a is given by [EL-[VIcos 6 Xa he a3 ‘Substituting for Lg and fy from (3:31) and (3.30) snto (3.28), the real power with armature current neglected becomes sind salve ESE ons 832 “The power equation contains an additional term known as there Equations (3.25) and (3.32) can be utilized for steady-state ar ceuit analysis, assuming & high X/R ratio, tne power factor approscties zero ana the quadrature component of current can often be negiested. In such a case shown that Xy takes on different values, depend “These reactances are vsually 3.5 POWER TRANSFORMER ‘Transformers are essential elements in any low voltages from generators t0 be rarset transmission. At the user end of the system, values mest surtable for dergo four or five transformations between generator and al ‘given system is likely to have about five times more KVA of ‘yansformers than of generators. 3.6 EQUIVALENT CIRCUIT OF A TRANSFORMER “The equivaient ciuit mode! ofa single-phase transformer s shown m Fis The equi consists of an ideal transformer of rt geste ‘elements which represent the imperfections of te real wansforner. An id former would have windings with zero resistance and 2 lossless, mfinwe pern ability core. The vottage Ey across the primary of the ideal transform tne rns voitage induced in the peimary winding by the mutual lux 6, This ts the portion of the core flux which links both primary and secondary coils. CIRCUIT OPA TRANSFORMER 65 2) = Rat jXy oA aor . sinysordal Ux @ = Brees cose voltage es 15 oan where Therefore, for an id 66 2, OENERATOR AND TRANSFORMER MODELS: THE PER-UNIT SYSTEM Ina real transformer, the reluctance of the core is finite, and wnen the secondary ‘current fy is 2er0, the primacy current has a finite value. Since at no-load, mace voltage By is almost equal to the supply voltage induced voltage and the flux are sinusoidal, However, because ofthe mt haractenstis of the foreo ‘and contains odd harmonics, certain three-phase connections of transformers. For the purpose of modeling, we assume a sinusoidal no-load eur- rent with the cme value of Iq, known as the no-load current. This current has & ux, known as the magnetizing current, © Set UP sging the induced voltage Ey by 90°, Im 18 as0 lag ‘90° Thus, this component can be represent The other component of the eddy-current and hysteresis losses in the core. Since im phase with E, and is represented by the cesistance: Rey the core flux. Since flux the induced voltage nnsformer with fimte reluctance, all of the flux 18 aot ‘both primary and secondary windings. The fux inas three companenis: nu primary leakage flux, and secondary leakage flux, The ieakage flux assoctated fone winding does not link the other, and the voltage drops caused by the leakage Aux are expressed in terms of leakage reactances Xv and Xz. Finally, Ar and Ro fare included to represent the primary and secondary winding resistances. To obtain the performance characteristics of a transformer nt {o use an equivalent cireuri model referred 10 one side of the iransiones. From Kirchhoff’s voltage aw (KVL), the voltage equation of the secondary side 1s Ey = Vet tah 338) From the relatonship (3.38) aeveioped for the ideal transformer, (ne secondary induced voltage and current are Ey = (Na/Ny)Ei and Je = ( see tuvely, Upon substitution, (3.39) reauces to Me (MPS a= 3h+ (a) Zaly = Vp + Zhi 4) where ta msixee (MY ets the KVL equation of tne secondary side referred to she pamary, cuit of Figure 3.9 ean be recrawn as shown 1 Figure 7.10, im the primary as would be produced in the see OFATRANSEORMER 67 A= Ry + iy poe Paneer meee Le 4 Ra S iXmg Bi —f ‘ FIGURE 3.10 eaetnanals Exact equvatert eet riered to te pi FIGURE 3.11 Approximate eae where saat vn in Figure 3,12, From 3.42) 68 3. GENERATOR AND TRANSFORMER MODSLS: THE PER UNIT SYSTEM Laz = Rent jXa FIGURE3.12 Approximate equivalent cient ferred to the scones. Power transformers are generally designed with very high permeal very small core loss. Conseaventty, a further approximation of the eq cit can be made by omitting the shunt branch, as shown in Figare 3.13. The alent circuit referred to the secondary 1s also shown in Figure 3.13, oy = Res + Xen eae Bag = Rag 5Xe2 vi Vj = Sve FIGURE S13 ‘Simplified ceuts refecred to one side 3.7 DETERMINATION OF EQUIVALENT CIRCUIT PARAMETERS ‘The parameters of the approxtmats equivalent suit and short-circi termunals of one be drawn from the suppl neglected, and the equivaient exc Since the secondary winding copper loss (resistive power loss NT CWUCUIT PARAMETERS 69) power ‘The st int elements Re 3.43) 7 Ban and (3.45) Theretore, the n flue, the exeiung. mnt branch can be ori leakage impedance 2, cor (erred secondary leakage impe 703, GENERATOR AND TRANSFORMER MODELS: THE PER-UNIT SYSTEM FIGURE 3.15 EBnervalen cut for the shont-eex Res and Xe may then be determined from the zelations ang (any Therefore, the equvaient leakage reactance ts = 20? ~ Res? 3.8 TRANSFORMER PERFORMANCE ‘The equivatent ciremt can now be used to predict the performance tues of the transformer. An important aspect «5 the transform en transformer efficiencies very from 95 percent to 99 percent, the higher being obtained from transformers with the greater ratings. The actuat a transformer in percent 1s given by output power = ei poner 49) = “put power and the conventional efficiency ofa transformer at n fraction of the full-tond power is given by nx $x PF 5 2.50) ~ ax Se PRX Path where S$ a three-phas ing equivalent T 3. GENERATOR AND TRANSFORMER MODELS: THE PER-UNIT SYSTEM ‘A program called trans is developed for obtaining the transformer perfor- ‘mance charactensties. The command trans displays a menu with tnree opto} Option 1 calls upon the function (Re, Xm] = troet(Vo, fo, Po) whick prompts the user to enter the no-Load test data and returns the shunt branch paramete Ze = trset(Vse, Isc, Pse) is loaded which prompts the user to enter the short test data and retums the equivate ‘whien prompts branch. This fu required data, ‘After the selection of any of the above options, the program promis {0 enter the toad specifications and proceeds to obtain the transformer p: charactenstics including an efficiency curve from 25 to 125 percent of Example 3.4 Data obtained from short-circust and open-ciret (60-Hz transformer are: sof 2 240-KVA, 4800/240-V Open-cirenit fest, Shorecircun low-side data gh-side di Vi = 240 = 1ST V Ig= 1A =50A A= ow 5c = 2625 W. Determine the parameters of the equivalent circuit ‘The commands trans dispiay the following menu PERFORMANCE 73 Selecs To roput transformer equivalent To quay Select number 2: menu — i Enter Transforaer ratea power Enter rated low Enter rated op: to Low sade or to hign sige — "ly P= ago to low side or recuced ansut neh ref. to HV sade Km = 30.000 onn brancn re 2 os on ret. tot side * 4 3.6000 onm 74 3. GENERATOR AND TRANSFORMER MODELS; THE PER-UNT SYSTEM ‘Transformer Efficiency, pf = 0.8 98.2, : 98.06 9738] Pane 978 | ord 972) Percent 97.0 1 96.3 568 ;9 “e080 00 120 14) 160 180 Ouiput Power, KW 5220 FIGURES.16 Efficiency curve of Examele 3. Seconaary load voltage = 240,000 ¥ Secondary loaa current = 100,000 A, at -36.27 degrees Current ref. to primary = 50,000 at ~36 Pramary no-load current = 0.518 A at -S3.1 Pramary amput current = 80.495 A at ~37 Primary input voltage = at Voltage regulation = Transformer efficiency = Maximum efficiency 1¢ 98.015 percent, occurs wath 0.80 pf At the end of this ana ine program menu 1s displayed 71ST RVR 3.9 THREE-PHASE TRANSFORMER CONNECTIONS ‘Three-phase power 1s transformed by use of extra high voltage (EHV) u may require 2 bank of three sin arrangements, id shipping -phase wansformers connected in unree-phase ASE TRANSFORMER CONNECTIONS 75 i vollages are in phase, The ¥-Y 4 = three-phase power mown as tne V or A-Y. This connection 1s 76. 3, GENERATOR AND TRANSFORMER MODELS, THE PER-UNIT SYSTEM 39.1 THE PER-PHASE MODEL OF A THREE-PHASE TRANSFORMER In ¥-Y and A~A connections, the ratio ofthe line voltages on HV and LY si the same as the ratio of the phase voltages on the HV and LV sides. Furtherm: there is no phase shift between the corresponding line voltages on the HV and LY sides. However, the Y-A and the A~Y connections will result ma phase shift ot 30° between the primary and secondary fine-to-line voltages. The windings are arranged i accordance to the ASA (American Standards Association) sui thi the line vottage on the HV side corresponding line voltage on the LV side by 30° rega shown in connection is shown in Figure 3.18. where Van is taken as reference. Let the V Ven peceeylan yy, FIGURE 3.18 10° phase sift im linetoline volages of V-a conection connection be the high vollage side shown by letter H and the A connec low voltage side shown by X. We consider phase a only and use subscript L fine and P for phase quantiues. If Ny 1 the number of tums on one phase the high voltage winding and Jy ig the number of urns on one phase of tne a= Nu/Ny = Var, ¢ voltage and phase veltage magnates 1s Vit = V8Vap Vu = Vee ‘Therefore, tne rato of the line voltage magnitudes for Y-A transformers Because tne core losses and magneuzaton cure ine order of | percent of the maxemum rat for power transforme: shunt impedance 1s neglect ‘senes so the senes From an only one of the ea approximation 10 the actual transformer vatues. FIGURE 3.20 (2) Two. winding transformer, () reconnected as an autotransformer From Figure 3.20(a), ie two-winding voltages and currents are rested by “MoM ya mne (3.56) and RUM. easy amc 37 where a 15 the tums ratio of the two-winding transformer. From Figure 3.20(b), we nave Vi = Vet Vi 3.58) Substtuang for Vj from (3.56) into (3.58) yields PestenPasat =yrmy 59) Yam Vata (3.59) Since V2 = Ve, the voltage re former becomes tionship between the two sides of an autotrans. vy = V4 Mb n= Vet 5AM = (14a), 360) or Van Gealte Ben 20. AUTOTRANSFORMERS 79 pe the transformer 1s ideal, the mmf d tbe e and opposite to the mmf produced by fy. As a result, we Nake = 3.62) From Kiechhof's law, fy = fp ~ fh, ane & becomes Nae) = Wt 3.63) or ips 3.64) Since A) = Iy, the © former becomes fe own sides of an aucorrans- 3.65) The ratio of the apparent gowe transformer, known as the power jormer co a rwoowinding 3.65) igher rating transformed 10 secondary vs known as the of the same raung, internal impedance power systems where the fer by a factor ihout being coupled ucted power Compares soxages of the greater than about chree, Example 3.5 A evo-winding trans'ormer 1s rated at 6 ated as a conventional ivo-winding tt 1s 0.96, Thus transformer 15 (0 a6 "e He. When oper 0.8 power factor, 1s 00. step-down au- vwhea used as an one MULL; LHe: PER-UNIT SYSTEM (©) Find tne efficiency with ine KVA loading of part (a) and 0.8 power facto ‘The two-winding transformer rated currents are: 60,000 f= Re 280 A Jy = 0.000 sy 1200 ‘The autotransformer connection is as shown in Figure 3,21 FIGURE3.21 Auto transformer connection for Example 35, (a) The autotransformer secondary current 35 {, = 250450 = 300 4 With windings carrying rated currents, te transformer rating is 3 = (1200)(300}(10~} = 360 KVA ‘Therefore, the power advantage of the autotransformer 1s (b) When operated as a two-wiading transfor losses are found from the efficiency formuta (60)(0.8) (Os) Fa, 7998 mee at Fullsload, 0.8 power Factor, the Solving the above equation, the 4 Prose = nercent lent impedance 1s primary, secondary, slormers in power jastormess and Y= connected autotransformers are provided with A-comnectea tertiary windings for harmonic suppression. 3.1L. THREE-WINDING TRANSFORMER MODEL Ifthe exciting current of a three-winding transformer is neglected, «draw a simple single-phase equivatent T-cireutt as shawn in Fig 1s possible to FIGURE 323 Ecquvalent circu of three-wansing transformer Three short-circust tests are cared out on a three-windin Np, Bq, and N; toms per phase on the three windings, respectively. T! are jn that sn each case one winding is open, one shorted, and reduced a cr applied to the cemaimung winding, The following smmpedances side to which the voltage 1s applied. Zou = mpedance measured in the primary eircutt with the secondary short-circuited and the tertiary open. Zu = impedance measured in the primary exrcuit with the tertary short Ciscunted and the secondary open. Zi = mpedance measured m the secondary circuit wit short-circuited and the primary open, primary side, we obtain Refernng Z! Tf 25, Zs, and Z, are the impedances of the tnree separate windings referrd to the primary side, then yy = Ey Beg Zou = Ze + Ze 3.6) Baa Bethe DL:TACE CONTROL OF TRANSFORMERS 83 zg Solving the anow SN 6.69) 0 may be fre: change a tap. A s TCUL transformer 15 connected. A TCUL, 84 3, GENERATOR AND TRANSFORMER MODELS: THE FER-UNIT'SYSTEM transformer may have by voltage sensing circuitry that automatically changes taps to Keep the system vol Such special transformers are very com- ‘mon in. modern power systems. Special tap changing gear are required for TCUL, transformers, and the position of taps depends on a number of factors and requires, special consideration to arrive at an optimum location for tie TCUL equipment. ‘Step-down units usually have TCUL in the tow voltage winding and de-energized taps inthe high voltage winding. For example, the high voltage winding equipped with a nominal voltage tums ratio pius four 2.5 percent to yield <5 percent buck or boost voltage. In addition to this, there could be pro: vision, on the low voltage windings, for 32 incremental steps of $ each, piving an automatic range of +10 percent. ‘Tapping, on both ends of a radial tcanstission line can be adjus’ pensate forthe voltage drop in the line. Consider one phase ofa three- rission line Wi former at the receiving enc of the line. A Figure 3.24, where ts and tp are the tap set the supply phase voltage refered to the high voltage side, and V; isthe load phase voltage, also referred to the high voltage side, The impedance shown mncindes the FIGURE3.24 ‘Acadia line with ap changing tanstormers at both ends line impedance plus the referred impedances of the sending end anc the ‘end transformers to the high voltage side. If V5 and Vg are the phase both ends of tne line, we nave FIGURES.5 outage praser diagram, 1. VOLTAGE CONTROL OF TRANSFORMERS 85 tween the component iMesh = | baal 1 8 will cesult in 372) Since Wj = ts sof Vf. and Vf becomes 3.73) Example 3.6 A three-phase tran: Ine load et eA ITY IRANOTURMER MUUELS) LE PER-UNUL SYSTEM The load real and reactive power pet phase are y= F(050)(08) =40 MW = 5(050)(0) = 30 Mvar ‘The source and the toad phase voitages referred to tne high voltage side arc () @)-3 From (3.75), we have ts= | ames = 108 pe CeO and. a tees n= Tog = 0.928 ou 3.12.2 REGULATING TRANSFORMERS OR BOOSTERS Regulaung transformers, also known as doosters, ae used to change ine votage ‘magnitude and phase angle at a certain point in the system by a sraall amount. A. booster consists of an exesting transformer and a semes transformer, VOLTAGE MAGNITUDE CONTROL, Figure 3.26 shows tne connection of a regu! ng transformer for pas Other phases have iden tapped, and the voltage o fiom iis applied to the primary of the senes transformer. The correspotding» age on the secondary of the series transformer 1s added fo the input voltaga Thus, the output voltage is Vila = Van + AVen (076) Since the voltages are m phase, a booster of this type 1s ‘The out By changing the switen from pos series transformer is reversed, 50. voltage. m 1 to 2, the polanty of the voltage 1€ Output voltage is now less tha ‘ONTROLOF TRANSFORMERS 87 ragonde cone PHASE ANGLE CONTROL, Regulating transformers are also used 10 imyected voltage 1s out ot phase have a pnase shifi ‘88 2. GENERATOR AND TRANSFORMER MODELS; THE PER-LNET SYSTEM Ve FIGURE 3.28 ‘Voltage paso dlagram showing phase shifting effect fer phase a output voltage can be made to lag or {ead the mput voltage, The advantages of the regulating transformers are |, The main transformers are free from tappings. 2. The regulating transformers can oe used at any intermediate pont in tne system, 3. The regulating transformers and tne tap changing gears can De iaken out af Service for maintenance without affecting tne system, 3.13 THE PER-UNIT SYSTEM. n of an interconnected power system having several different es the cumbersome transformation of all impedances age level. However, power system engineers have devised the per-ur that the various physical quant fre expressed as a decimal fraction or multiples of base qua system the different voltage levels disappear, and a power network involving generators ‘wansformers, and lines (of different voitage levels) reduces to a system of simple impedances, The perunit vaiue of any quantity 18 defined as actual quant base value of quanniy Quantity in per-umit = (3.78) Su THEPERUNIT SYSTEM 89 For example, Bie’ (3.80) The 3.82) voitage 6.84) 3.85) vidual generators and transformers, as supplied by the man In terms of percent or per-unit quan Faungs. The impedance of ima sysiem, the remaining voltage bases mined by the vanous transforme! if on a low-voltage side of a 34.5/115-K¥ tra Selected, the base voltage on the high-voltage side must be 120 KV. Nermaily, we iy to select the voltage ba: ‘same as the nominal valves Let Zeid be the per-unit impedance oa base Vg! which is expressed by are no longer indepencent; they are dete the power base Si und the votage Expressing Zn to a new power base ‘and a new voltage base, results in the new per-umit impedance From (3.89) and (3.90), the relationship between the old and the new per-unit vale ues 1s anew g 8.9) If the voltage bases are the same, (3, apse = gaunSe (3.92) for analysts are described below. ‘The agvantages of the per-unit sy The pecuntt system gi plate magnitudes of various and impedance. ume general ype based on the fF the raung of the equipment. id the power and vol ified since the factors fedance diagratn 1s obtained for 12 diagram ofa three-phas sho common base of 100 MVA and 22 diagram with ail impeaances 1 carer's data for each & Figure 3.29. Select fe, Draw an impedance mice marked in per-urit. The, many MCHANGBOP BASE 93. 92 3. GENERATOR AND TRANSFORMER MODELS: THE PER-UNIT SYSTEM e Since generator and (ransformer voltage Dases as their rated values, TR thetr persue reaetances en 2 100 MIVA base tL uni 7 C4 220 k¥ i oD ie M } 6 te uiez YC |] tesa 110 kV v > C FIGURE 3.29 ‘Onestine diagram for Example 37. 66.5 MVA and 10.45 KY, From (3.91) the CG. SOMVA 22kV = X=18% Ti: SOMVA 22220KV X= 10% Ts. 40 MVA X=6.0% Ty: 40MVA X 264% Ty: 40 MVA X=80% M: 66.5MVA 1045kV X=18,5% ‘The tnree-phase toad at bus 4 absorbs 57 MVA, 0.6 power factor iagging at 10.45 KV. Line 1 and line 2 have reactances of 48.4 and 65.43 2, cespectvely First, the voltage bases must be determined for all sections of the ‘generator rated voltage 15 given as the base voltage at bus 1. This fixes ‘bases for the remaiming buses in accordance to he transformer tums base voltage Vigy on the LY side of T, 1s 22 KV. Hence the base on 1ts HV side 1s 2 per-unt 200 Van = 22555) = 220 kv ‘This fixes tne base on the HV side of Tat Ves = 220 KY, and on its LV side at u fps = 204) = Vou = 220( 555) = 11 kV Similarty, tne voltage base at buses 5 ana 6 are 10 oy Vas = Vas = 22%") = 110 wv ‘The base impedance for the load is = Ot Fea Sala 9 ‘Therefore, the load impedance in per-unit is 1.1495 + 1.53267 Tar ‘The per-umnt equivalent circuit is shown in Figure 3.30. Baggy = = 095+ 51.2667 pa 4 50.20 50.10 ois 4 j0.54 rngedance diagram for Example 2.7 Example 3.8 ‘The motor of Exampie 3.7 operates at full-load 0.8 power factor leading at 3 term nal voltage of 10.45 kV. (a) Determine tne voltage at the generator bus bar (bus 1) (>) Determine the generator and the motor internal ems, (2) The pernit voltage at bus 4, taken as reference is Vy = O88 Lo 950° pu n ‘The motor apparent power a 0.8 power factor leading is given by Sn = 83/3587 pa 100 CHANGEOP BASE 95 96 3. GENERATOR AND TRANSFORMER MODELS; THE PER-UNIT SYSTEM 3.2, A 60-MVA; 69.3-KV, three-phase synchronous generator has a synchronous reactance of 15 iiphase and negligible armature resistance. (a) The generator 1s deliverng rated power at 08 power factor lagging at the rated terminal voltage to an infinite bus bar. Determine the magestude of the generated emf per phase and the power angle 5. (b) If the generated emf is 36 kV per phase, what 1s the maximum three- phase power that the generator can deliver before losing its synch ) The generator is delivenng 48 MW to the bus bar at the raved vol field current adjusted for a generated emf of 46 KV per phase. D: ‘mint the armature current and the power factor. State whether pov ing or teading? 33, A 24,000-KVA, 17.32-KV, 60-He, three-phase synchronous generator has & ied load. 0.8 power factor ltage of 17.32 kV, Determane the excitation voltage per phase (b) The exestauion voltage 1s maintained at 13.4 KV/pnase and the lera voltage at 10 kV/phase. What 1s the maximum threc-pnasc real power th te generator can develop betere pulling out of syncaromsin? (©) Determine the armature current for the condition of part (bi 34, synchronous generator nas a a quadrature-anis reactance of 9.333 9, ‘The armature resistance (a) Referring to the phasor diagram of a salient-pole generator shown in Fig ure 3.8, show that the power angie 6 1s given by mann ( _Xallabeosd fae (y pees) (©) Compute tne toad angle 6 ana the per phase exci the generator delivers rated MVA, 0.8 power factor lagging to an ar of 34.64-4V (c) The generator excitation voltage is Kept constant at che value found im part (b). Use MATLAB to obtan a plot of the power angle curve, ie, equa tuon (3.32) over a range of 6 = 0:0.05;180° Use the command Pmax, «= max(P); dmax = dik), to obtain the steady-state maximum po tnd the corresponding power angle dinax. LIM.CHANGEOF BASE 97 02+ 70452 ieee nag mere EPC 8.002 + 70.0045 1 pooneneree in tia i tf tm | i vi 1000 rae a é va | save 400/240-V shown in Figure 3.31 Determine the eur BI. 38, 39, 3.10. 3. MERRRALUR ANU LKANSFURMEK MUDELS, UTE FERCUNIE SYSTEM (©) Verify your answers by running the trans program in MATLAB and obtain the transformer efficiency curve, A two-winding transformer rated at 9-kVA, 120/90-V, 60-HZ. has a core loss of 200 W and a full-oad copper loss of $00 W. (a) The above teansformer iso be connected as an auto transformer to supply load at 120 V from a210-V source, What KVA toad can be supplied without exceeding the current rating of the windings? (For this part assume an ideal transformer) (©) Find the efficiency with the VA foading of part (a) and 0.8 power factor, Tree identical 9-MVA, 7.2-KV/4.16-KV, single-phase transforrners are con nected in wye on the high-voltage side and deita on the low vo e Series impedance of each transformer referred to the high side 1s 0.12 + 40.82 9 per phase. The transformer supplies a balanced thuce. Phase load of 18 MVA, 0.8 power factor lagging at 4.15 KV. Determine the Tine-toline voitage atthe high-voltage tezminals of the transformer A 400-MVA, 240-KV/24-KY, three-phase ¥-A transformer has an series impedance of 1.2 + 36 9 per phase referred to the high-voltage ‘The tansformer ts supplying a three-phase load of 400-MVA, 0.8 power factor lagging at a terminal voitage of 24 KV (line line) o Side. The primary 1s supplied from a fecder with an impedance of 0.6 + J1.2 © per phase, Determine the line-to-line voltage at the high-voltage ter- ‘munais of the transformer and the sending-end of the feeder In Problem 3.9, wi impedances 1 ¢ at tne high-voltage termmnais ofthe transformer andi tne sending feeder, ‘A three-phase, Y-connected, 75-MVA, 27-kV synchronous gene synchronous reactance of 9.0 9 per phase, Using rated MVA and base values, determine the per-untt reactance, Then refer this per t02 100-MVA, 30-KV base. ‘A 40-MVA, 20-kV/400-KY, single-phase transformer has the followine se nes impecances: 2, = 09 + 71.8 Mand Z = 128 + 72880 Using the transformer rating 0s base, determine the per-anit impedance of the transformer from the okrmue value referred to the low-voltage side. Compute the perunit impedance using the onmic valve referred to the high-voltage side, 3.13. Draw an impedance diagram 3.32 showing all impedances sn pet as the voitage base for gow n below. 90 MVA, 80 MVA, 80 MVA 90 MVA AML CHANCECEBASE 99 «power system shown in Pigure (00-MVA base. Choose 20-kV 20Kv 20/200 kV 200/20 kV 18kV 200 kV 200kV 100 34s. 3. GENERATOR AND TRANSFORMER MODELS: THEPER-UNTT SYSTEM G 80 MVA, 22kV X=u% qT: ‘50 MVA 22/220kV X = 10% Tt. 40MVA 220/22 kV X = 6.0% T. 40MVA iloKY X= 6.4% 220 kV X=1210 Line 2: HOkV X= 42352 M: 6885MVA 20KV X= 225% Load: 10 Mvar — 4kV—— connected capacitors The three-phase ratings of the taree-phase transformer are Primary: Y-connected 40MVA, 110 kV Secondary: Y-connected 40 MVA,22 kV Teriary: —A-connected 15 MVA, 4 kV ‘The per phase measured reactances at the terminal of a winding with the second one short-cireuited and the third open-circuited are Zs = 9.8% 40 MVA, 110 kV/22 KV Zp = 7.2% 40 MVA, 110 KYA KV 2a = 12% — 40 MVA, 22kV/4 KV Obtain the T-cireuit equivalent impedances of te three-winding transformer to the commen 100-MVA base. Draw an impedance diagram showing al) impedances in per-unit on a 100-MVA base. Choose 22 kV as the voltage base for generator, ‘The theee-phase power and line-line ratings of the eleetne power system shown in Figure 3.34 are given Deiow. FIGURE 334 (One-line agra for Problem 3.15 Gi GOMVA 20KV 0 X=9% Ti SOMVA 207200 kV X= 10%, T, SOMVA 20020KV XX = 10% Mo @2MVA KV _X=8% Line: 20KV J = 120 + j2000 (a) Draw an impedance diagram showeng all impedances in pet 100-MVA base. Choose 20 KV as the voltage base for generator, S16. CHANGEOF BASE 101 wer factor lagging ata minal voltage and the internal 316, The one-line diagram of a 3.35, Impedances are marked sn pe Sp = 15.93 MW ~ 1d to hold th 100-MVA, 400-KV base. The is Sy = 77MW bbus 3 at 40020° kV. Working load at bus 2 346 fem 1s as shown in Figure base of 100 MVA. 23/115, atbus 2is S: = 184.8 CHAPTER, 4 TRANSMISSION LINE PARAMETERS 4.1 INTRODUCTION “The purpose of a transmission network 15 to transfer electric energy from se ang unts at vanous locations to ihe distribution system which uiumatety the load. Transmission lines also interconnect neighbonng utilities which perm riot only economic dispatch of power within eons during normal conicions, ns aiso transfer of power between regions during emergencies. Al transinussion lines in a power system exhibit the electrical properties of resistance, inductance, capacitance, and conductance. The inductance and capac: itance are cue to the effects of magnetic and electne fields around tne conductor, ‘These parameters are essential for the development of the transmission line moxt- cis used in power system analysis. The shunt conductance accounts currents Flowing across insulators and ronzed pathways im the air. The ‘currents are negligible compared to the current flowmg m the transmis and may be neglected ‘The first part of this chapter deals with tne determination of inductance anc capacitance of overhead fines. The concent of geometric mean radius, GMR anc geome:ric mean distance GMD ae discussed, and the furction GMD. GMRL, 102 MR and GMD. This furteton of single-circuit or GMRC1= gm is dev 4.2 OVERHEAD TRANSMISSION LINES .nd usually shield wires, from a tower usually fay, Steel tow- owers have fen width che present a solu- invoived with overnead that make the use of tuon to some of the sst0n kines, th 104 4. TRANSMISSION LINE PARAMETERS The selection of an economical voltage level for the transmission ‘on the amount of power and the distance of transmission. The ice to- gether with the selection of conductor size 1s mainly a process of weighing 1? losses, audible noise, and radio interference teve fixed charges on vestment, Standard transmission voltages arc the Unitea States by the American National Standards Institute (ANSI). Transmission voltage lines op erating at more than 60 kV are standardized at 69 KV, 115 KV, 138 KV, 161 kV, 230 kV, 345 KV, 500 kV, 765 kV voltages above 230 kV axe usually referred to a5 extra-high voltage (EHV) and those at 765 kV anu above are referred to as IV). The most commonly used conducior ‘materials for high reinforced), AAC loy-remnforced), The reason for their pop- ¥y 1s thetr low relative cost and high strengit-to-wergnt ratio as compared to aluminum 1$ in abundant suppiy, while copper Xf the most commonly used ACSR conductors 8 of other conductors can be found 1n conde (oF manufacturer's literature. The conductors are stranded (0 have flex ACSR conductor consists of a center core of steel strands surtounded by layers of aluminum as shown in Figure 4.2. Each layer of strands 1s spiraled in the opposice direction of its adjacent layer. This spiraling holds the strands in place, FICURE 42 ‘Crossectonal view ofa 24/7 ACSR conductor, Conductor manufacturers provide the characte 4 solid round conductor # inch 1m diame have been assigned to each condix At voltages above 230 kV, In addition, code words (bia names) f for easy reference. ‘S preferable to use more than one conductor Ne chee laetors LS LINERESISTANCE 105 4.4 INDUCTANCE OF A SINGLE CONDUCTOR A cunrent-carrying conductor produces a magnetic field around the conductor. The ‘magnetic flux lines are eoncentic closed circies with direction given by the cei. hand rale, With the thumb pointing inthe di Lat 3) where A = flux linkages, in Weber turns Consider a tong round conductor with radius r, carrying a curcent fas shown in Figure 4.3. FIGURE43 Flux linkage ofa long round conductor, The magnetic field intensity He, around a circle of radius 2,15 constant aod tangent to the cicle, The Ampere’s law relating H. to the current [1s given bv [Cted=t 4) A I, Heo Fe a5 Bez 7; whore Jy s the current enctosed at radius 2. As shown m Figure 43, Equation (4.5) 1s all that 15 required for evatuating the flux linkage A ef a conductor, The inductance of the conductor can be define skages unternal and externa fo tne conduc 44.1 INTERNAL INDUCTANCE A simple expressi cut the condvetor eros 46) 7 'Y pig, the magnete flux. (438) x 10°7 Hin, meter length of 2 permeat do for a small re 49 center to radius on mz2/mr? of (6.10) (4.12) 108. 4. TRANSMISSION LINE PARAMETERS 442 INDUCTANCE DUE TO EXTERNAL FLUX LINKAGE the conductor at radius > 7 as shown in Figure 44 se encloses the entire current, Ze = T and in (45) Ie 8 cy at radius x becomes Consider Hy extem: pol = oe = 413) Be = polls = 52 Tinkage between Dy and Ds Since the entire current J ig inked by the flux outside the conductor, tte fage dys pumencally equal to the flux dz. The differental flux az fora small region of thickness dz and one meter length of the conductor ss then given OY Z dy = doe = Bede 1 = oar adi Bre “The external flux Jinkage between (wo points D, and Dy 18 found by inkegraung Az from Dy to Da ew = BE Las bs erin 22 wom is x 10 inp: ‘ov nductance between two points external to a conductor 1s then (4.16) 4.5 INDUCTANCE OF SINGLE-PHASE LINES, "EOF VINGLE-PHASELINES 109 two solid round conductors are current J) referenced ese currents set “ay (4.18) Let r, = rye 4, the inductance of conductor 1 becomes Ly =2x 10h Fe2xn? Hin (4.20) Similarty, the inductance of conductor 2 is D Ly =2x107 Fran tn® wim (2) 4 If the two conductors are id Inductance per phase per meter length of th 1 = t= rand Ly = Ly = L, and the ine s given by Examination of (422) reveals that the first term is only a function of the conductor radius. This term is consideredi as the inductance due to bath the internal flux and that external to conductor | to a radius of fm. The second term of (4.22) is depen- ‘dent only upon conductor spacing. This term 1s known as the inductance spacing factor. The above terms are usually expressed as inductive reactances at 60 Hz and ‘re available in the manufacturers table in English units, same inductance as the actual conductor with radius r. Gil to as geomeinc mean radius and will be designatea by D,, Thus, Der phase in millihennies per kilometer becomes D =02In = 23) £=02Ia 5 mia . 4.6 FLUX LINKAGE IN TERMS OF SELE- AND MUTUAL INDUCTANCES ‘The sens inductance per phase for the above single-phase tw0-9 expressed in terms of seifinductance of each conductor and thei tance. Consider one meter length of the single-phase circuit sd by dot symbols as shown tn Fi ‘The flux linkages 1 and Ao are given by Ms Luh t+lieh Ars Lah +inh (2a 416, BLUXLINKAGE IN TERMS OF st h Lu Lia fag |e cunt as = Since fy = hr, we have LINDUCTANCES awed a5 99 magnencally coyotes wu (4.25) ang equivalent (428) (4.29) a7. NDU 47 INDUCTANCE OF THREE-PHASE S3IASE TRANSMISSION LINES 113 ‘TRANSMISSION LINES 47.2 ASYMMETRICAL SPACING 471 SYMMETRICAL SPACING 2 al spacing of conductor: Consider one meter fength of a three-phase line with three conductors, each with ig of conductors Satins 7, syrnmetneally spaced in a tnangular configuration as shown yn Figare 4.7 Ie ‘ és FIGURE 47, “Three-nas line with syinmetnea! spe. ‘Assuming balanced three-phase currents, we fave [tht he=0 (4.30) oom (425) he total fx inkage of phase conductor i eee eeeeeeeee eee paren (ambrnnzenmg) an Subst fr fy + f= =a de = 2107? (Io 7 D = 2x107hinS 32 Because of symmetry, Xy = Ae = a, and the three inductances are sdeniical ‘Therefore, the inductance per phase per kilometer lengt D La 02in 5 mien 1433) where r! 15 the geometric mean radius, GMR, and 1s shown by Ds. For solid ound conductor, D, = re for stranded conductor Ds can be evatuated from {4.50}, Companson of 4 (4.23) shows that anductance per phase for & tnree-phase circuit with equilateral spacing 1s the same as for 0 single-phase cireu conductor of 2 1144, TRANSMISSION LINE PARAMETERS where the operator a = 1120° and a? = 1/240°. Substituting sn (4,34) results m Muay (iad ty) ain = Ba 2x 10" (Int rnp, d» (ain Leg y= =2x107 (om sagt ooo OB Dar aXe 7 (In 2 een in tex a2 10 (#ings+emps Examination of (4.38) tne pase incuctances are 1 contain an imaginary term dye to the mutual inductance. 47.3 TRANSPOSE LINE [A per-phase model of the transmission line 1s required in most power system ysis, One way to regain symmetry in good measure and obtain per ts to consider transposition, This consists of interchanging the phase con every one-third the length so that each conctuctor 1S moved To Occupy the next phy teal postion ina regoar sequence. Such a tanspostion arrangement 1s shown 1 Figure 4.9. FIGURE 49 ‘transposed three-phase line Since in a transposed fine each phase takes al three p inductance per phase can be obtained by finding the average value of (4.3 iat byt be tat = (439) Noting a +o? = 12120% + 12240? = —1, te average of (4.38) pecomes 2x 10-7 ! (0 4 vine in ge = Be "Dm = 2x10 or the mduetance per phase pe ‘EOF COMPOSITE CONDUCTORS LIS (4.40) (4.41) (4.42) tance of one phase 1164, TRANSMISSION LINE PARAMETERS 2 Q e FIGURE 4.10 Single-phase ine with to composts corduciors. denna wot (In sin pot +5) wad (ingh wingheein che + or Ye = 2x 10-77 In ¥ (4.43) The inductance of subsonductor as La = Pesan x 107 in (4.44) dp Using (4.29), the inductance of other subconductors in are similarly obs For example, the mductance of the subconductor a z : [y= Pe = nx 10-7 In (45) Tn VDaaDna Doe “The average inductance of ny one subconductor in group 28 Ly = Lot Lot betes t bn er Since all the subconduetors of conductor = are electncally the mductance of x will be Tyga bee a bat bet let tl, en substicuting the values of La, Dy, Le,-++, Ln in (4.47 GMD Gun, 2 =2x 107 In Himeter (4.48) were (DusDut--* Dan) as es between ni strancs root of the product © from each strand tot rts then larly obtained, The geomett xe mean distance GMD, hor Exampte 4 A stranded 118 4, TRANSMISSION LINE PARAMETERS =r (m2 (2)8 (3)F 2)? = 2.767 ‘With a large number of strands the caiculation of CMR can become very tedious, ‘Usually these are available mm the manufacturer's data 48.1 GMM OF BUNDLED CONDUCTORS ‘auctors symmetric conductors within a bundle are separated at frequent intervals by spacer-damper ‘Spacer-dampers prevent clashing, provide camping, and connect the subconductor oo ] a add a SoH Gad Examples of bundled srangerens, The GMR of the equivalent single conductor is obtained by using (4.50). D, 1s the GMR of each subconductor and d isthe bundle spacing, we have for the two-subconductor bundle sn for the three-subconauctor bundle Di= PD. x ax d= YDxe for the four-subconductor bundle 49 INDUCTANCE OF THI DOUBLE-CIRCUIT LINES inductance per phase, To dt the GMD betwee (4.56 440, 4, TRANSMISSION LINE PARAMETERS where Dt isthe geometric mean radius ofthe tuned conductors given by ¢ (4.53). The equivalent geomeinc mean radis fr calculating the pee phase induc tance to neural is CMR, = YDsaDseDsc asp ‘The inductance per phase in millihenries per kilometer is emp = 02a re makin (4.58) 4.10 LINE CAPACITANCE ‘Transmisston line conductors exhibit capaciiance with respect to cach other cue vo the pot ifference between them, The amount of capacitance deeween cor ductors is a function of conductor size, spacing, and height above ground. By def ‘muon, the capacitance Cis the ratio of cnarge 10 the voltage V, grven by =i 4.59) cae 14.59) Consider a tong round conductor with radius r, camying a charge of per meter length as shown in Figure 4,14) coulombs FIGURE 4.14 Blesine felt around a tong round conductor The charge on the conductor gives nse to an electric field with radial flax fines. The total electric flux is numerically equal to the value of charge on the conductor. The intensity of charge and is termed electric surrounding the conducior are «% flux density. From Gaus's i flux densi¢y uta cylinder of red as the force per v B, Concentne cylin ices and have the same elect of the conductor, the elect (4 The etactne i mensity E aa she perm ang (4.60) 10 (4 61) equal to 8.85 » 10°? Fim, Sub, ta “The poten (he work done in moving 3 u se field! produced by Day ts defined as Lisung AML CAPACITANCE OF SINGLE wo tong solid rou The ovo conductors 12 of gr coulombs/me meter. The presence of the s ‘nrst conductor, The distance 1d the height of conduct mt effect 15 small and ice ofthe conductors the voltage bene tha 120.4. TRANSMISSION LINE PARAMETERS where D¥is the geometric mean radius of the bundled conductors given by (4 (4.53). The equivalent geometric mean radius for calculating the per-phase tance to neutral is, GMR, = YDsaDsaDsc usp ‘The inductance per phase in millihennes per kilometer is, cup i =02In mi 4.5 Em 02in cage mm (58) 4.40 LINE CAPACITANCE ductors isa function of conc mton, the capacitance C is the @ cat $ a long round conductor with radivs r, carrying a chase per meter iength as shown in Figure 14. und corer radial flux hasge on Une \. 8 conductor gives nse co an eletric field \ {ne flux 1s numencally equal to the valve of Pal HASCLINES 121 the force per unit ect as 2, Concentric eylinders 1224, TRANSMISSION LINE PARAMETERS a 2 © 9 FIGURE 4.15 Single-ohase (wo-wae ine, Now assuming only conductor 2, having a charge of ga, the voltage between con: ductors 2 and 11s 2 Yantar) = Fre5 Since Viagq = ~Yeuimn We nave Miata) = pe (463) Vite) = Seg "DB 65 From the principle of superposition, the potential difference cus to presence ot both charges 1s Va=Vi aim = 5 seme 5.66) + Vine = Seg OF * Bae tte For a single-phase line 92 = ~q1 = ~9, and (4,66) reduces 19 Va= tm? Fim «on me 7 From (4.59), the capacitance between conductors ss x0 eee Fim (4.68) Equation (4.68) gives the line-to-line capacitance between the conductors. For the purpose of transmission line modeling, we find it convemtent to define capacitance C between each conductor and a neutral as ifustrated im Figure 4.16. Since she FIGURE 46 TMusiation of capacitance to neural = 8.85 « 1C 412 POTENTIAL DIFFERENCE IN MULTICONDUCTOR CONFI 1 FIGURE S17 Multicondoezar contgu Assume that the distorion effect 18 2 distributed around the conauctor, with the ators Using supesposition and (4.53). 5 due to the presence ofall charges 38 When & = 1, Di namely tts 12 WC = 20 rp, oF (4.69) to pF per kilometer, we have (4.70) a comlombs/meter as and the charge 1s uniformly any in conductors + and y 47 14 4. TRANSMISSION LINE PARAMETERS: 4.13 CAPACITANCE OF THREE-PHASE LINES Consider one meter tength of a three-phase line with three long conductors, exci, ‘with radius 7, with conductor spacing as shown Figure 4,18. a ‘Theee-ohase transmis Since we have a balanced three-phase system Get m+ I= 0 (473, We shall neglect the effect o the shueld wires, transposed, We proceed w of the pe and 6 for each section of transposition. Appiying (4.72) co the first sect (374) Silat, for che secon seston of he ceanspostion, we ave 1 De: r Dd Voon= sic (un 22 range romZ2) urn ne ( 7 TeinD Tend, ane forthe fast section L D Vaugin = 5 Sain qin Se (376 0th = 5 wage teas v DaDade a, ar (DPePis) ombsat) (4.78) ym arguments and ts (4.79) (4.80) sp (4.82) fetiy as) “Theretore. 48) 2) the api (4.85) ‘This 1s of ine si spacing, Fi ove iree-ph three-phase spacires, 1264, TRANSMISSION LINE PARAMETERS 4.14 EFFECT OF BUNDLING ‘The procedure for finding the capacitance per phase for a three-phase tcansposed with bundle conductors follows the same steps as the procedure in Section 13. The capacitance per phase 1s found to be c= 2a 63 ing ae ‘The effect of bundling is to introduce an equivalent radius 7°. The equivatent ra- dius r* is similar to the GM. (geometnic mean radius) calculated e: ‘inductance with the exception that radius r of each subconcuetor 1s us Daf dis the bundle spacing, we obtain for tne two-sudconductor bundle 4.38) ra Vex e 4.39) for the four-subconductor bundle os 1090 x ae (4.90) 4.15 CAPACITANCE OF THREE-PHASE DOUBLE-CIRCUIT LINES calculated ana tne per phase equrvaent cap ‘ance to neutral is obtained t0 be 2reo cs Fim 91) in HE or capacitance to neutral in yF per kilometer is 0.0556 C= gap “ram (492) ‘The expression for GMD 1s the same as was found for inductance calculation and 's given by (4,55). The GMRe of each phase group 1s similar to the GMRz, with the excep equations; (4903) given by (4.88) ~ 3 per-phase capact OMRe = YFG (49a) 4.16 EFFECT OF EARTH ON THE CAPACITANCE the presence of earth ntroaueed by Keivin. To stead! a8 the shield wires soul be considered. Example 4.2 3924 in, Bundle per kilometer of A 500-KV three-phase transposed line is composed of one ACSR 1, 272,000- ‘mil, 45/7 Bittem conductor per phase with horizontal conductor configuration as shown in Figure 4,19, The conductors have a diameter of 1.345 in anc a OMR of (0.5828 in. Find the inductance and capacitance per phase per kilometer of the line, a ° « Di = 70 FIGURE 4.19 Concicor layout for Example 4.2 Conductor radius 1s 7 = 33 GMD is obtained using (44 GMD = YS X3H RT = 44.097 fe From (4.58) the inductance per phase 1s t = 0.086 ft, and GMR,, = 0.5328/12 = 0.0444 t. vm | 44.007 { 7 hem oo ~' om L=02In reduction im the and from (4.92) the capacitance per phase 1s 0.0556 for the computa: 155 I! C= ary = 0.0083. Fk | nee | Example 4.3 The line m Example 4.2 1s replaced by two ACSR 636,00 24/7 Rook Conductors which have the same total crass-sectionat area of aluminum 23 vit Bittern conductor, The line spacing as measucea from the center of the bunele is the sare as before and is shown in Figure 4.20. « 6 e 20 a0 0,0 TN Dy = 99 = a5 ! See FiGuRe 420 Conductor layout for Bxamoie 43, A504, TRANSMISSION LINE PARAMETERS e ° e oo oo oo 29 010 010 BT Dap = abst Day = 44.5 \ Dig = 89— ! Figure 421 (Conductor yout for Exampie 4.4. ‘The command acsr displays the conductor code name and the area in emis for the ACSR conductors. The user 1s then prompted to enter the conductor coce name ‘within single quotes, Enter ACSR code name within single quotes -> ‘rail’ Al Area Strana Diameter GYR Resistance Oha/ka Ampacity cmil Al/St cm cm 80z 25C 60Hz SOC Ampere 954000 45/7 2.959 1.273 0.0624 0.0683 1000 ‘The following commands MD, GMRL, CNRC] = gna; £0. 2Log (GMD/GMRL) 4 aA/en Eq. (4.58) C= 0.0556/log(GMD/amaC) 4 macro F/km Eq. (4.92) resuit im Number of three-phase circuits Entez Single-careaat i Double-carew.t vertacal configuration 2 Double-carcust norazontal configuration 3 To quit a Select nunver of menu —+ 1 Enter spacing unit within quotes Enter row vector (D12, D23, D13 Cond, size, bundle spacing unit: Conductor diameter 1a cm = 2.959 Geometric Mean Radius in cu = 1.173 Wo. of bundled cond. (enter i for single cond.) = 4 Bundle spacing in en = 46 GMD = 56.06649 fe GMRL = 0.65767 ft L = 0.8881 C= 0.0827 GMRC = 0.69696 ft | | CAPACITANCE 131 Example 4.5 composed of two ACSR, th vertical conductor con- figaration as shown in aGMR of 0.564 n. The tance per phase per ki oes =u Hy=Tm 65m "eS, 125m 90 Eq. (4.58) vo Eq. (4.92) result Nanber of taree-pnace < Single-carcaae Double-eazeait ver Douole-careat nor To quit Select nunoer o: aenu —~ 2 Cizevat Arvangosonts CD) abe-e™bea’ (2) avc-a'b'c Entor (i or 2) Enter spacany Enter row vecto: Enter rov vecto: 1324, TRANSMISSION LINE PARAMETERS Conauctor diameter in anch = 1.427 Geometric Mean Radius in 2nch = 0.564 No. of oundlea cond. (enter 1 for single cona.) = 2 Bundle spacing in inca = 18 GMD = 11.2136 = MRE. = 4.18731 a L = 0.4491 C= 0.0254 GYRC = 1.25920 m Example 4.6 A 545-KV douible-ireuitenree-phase transposed line is composed of one ACSR, 556, 500-cmil, 26/7 Dove conductor per phase with norizontal conductor configu ration as shown in Figure 4.23, The conductors have a diameter of 0.927 im and a GME of 0.3768 m. Bundle spacing 1s [8 in, Find the inductance and capacitance per phase per kilometer of the line, The followeng commands 7 : G- 8m GQ 8m -G 51-98 8m Ge mH FIGURE423 ‘Conauctor layout for Example 4.6, (oMD, GMRL, GRC) = gaa; L=0. 2610g (GMD/GHRL) © = 0.0556/Log (GMD/GHRC) % oH/em Eq. (4.98) % micro Ffln Eq. (2.92) result in lunber of three-phase circuits Enter Binglescarcuit i Double-circuit vertacal configuration 2 Double-circwit norizontal configuration 3 To que o Select number of menu > 3 Cireurt Arrangenents CD aber bec (2) ave-c'bia’ Enter (1 or 2) + 1 Enter spacing unat wztnin quotes *n’ or ‘ft! — a’ Enter row vector (Di2, D23, $13] = [8 8 16) Enter distance vetwaen to circuzts, S11 = 9 4.13. MAGNETIC HELD INDUCTION 133 cond. size, bundle spacing unit: ‘ca! or ‘in’ > fin Conductor diameter in anca = 0.927 Geonetric Mean Radius im inch = 0 No, of bundles cond. (enter i for 5. GHD = 14.92093 m GYRE = 0.48915 = 1 = 0.6826 c = 0.0168 cong.) = 1 GMRC = 0.84951 2 7 MAGNETIC FIELD INDUCTION tne brolog wien effects are the Example 4.7 i hots ) = OK 134 4 TRANSMISSION Lie PARAMETERS a ° e Q 36m -—Q- 36m De: Dea Da 4m a 8 112 mi ricurr 424 Conductor layout for Example 6, ‘Total flux linkage between conductors 1 and 2 due to all currents is Daa Dea Ata = 0.24 in 28 4.0.21, m 2 O2igin 52 +0.2iIa 52 mb For positive phase sequence, with J, as reference, Ic = [£~240? and we have i Der oy, Dea saa 02h (in Be 12-200» 22) tin With f, as reference, the instantaneous ffux linkage 13 Aaalt) = V2 [Araleos(ut +a) ‘Thus, the mduced voltage in the telephone line per kilometer length is dol!) v= = V2) Ara] cos(wt + a + 90°) dt ‘The rms voltage induced in the telephone line per kilometer is V = ulrrg|da + 90° = jwhie From the ciromts geometry Da = Da=(S +4)? =5 m Dey = Da = (42403 ‘The total flux finkage is 3), 88 5 0.2 x 20020" In 23 + 0.2 x 2002-240" in & ‘ 5 58 = 102832 = 50° mWWivien Aa ‘The voltage inauced in tne tetepnone line per kilometer is 0.2834.~30°)(10"7) = 3.88260" Vim V spur = 3! 4.18 ELECTROSTATIC INDUCTION pamary ca ‘The human boxy 1s aif objects im the field of the The current den known to be cine strength sductor surtace, Corona produces po\ we and radio and 15 in foul weather, Radio lower the cleaned penodical ig the cond “The power loss assoc ywever, under 1 feakage be 136 4. TRANSMISSION LINE PARAMETERS PROBLEMS 4.1. A solid eylindcal aluminum conductor 25 km fong has an area of 336.400 circular mils. Obtain the conductor resistance at (a) 20°C and {b) 50°C. The resistinty of aluminum at 20°C is 2.8 x 10-8 O-m, 4.2, A transmission-line cable consists of 12 identical strands of alurmintim, each 3 mm in diameter. The resistwity of aluminum strand at 20°C 1s 28 x 108 S-m, Find the 50°C ac resistance per km of the cable, Assume a sk ‘effect correction factor of 1.02 at 60 Hz. 4.3. A three-phase transmussion line 15 designed to deliver 190.5 MVA at 220 kV over a distance of 63 km. The 2.6 percent of the rated line MVA. If the resistivity of tne con is 2.84 x 10°# O-m, determine the requized conductor diam ‘conductor size in circus mils. 44, A single-phase transmission line 35 kin long consisis of two solid round con- ductors, each having a diameter of 0.9 om. The conductor spacing is 7.5m. Calculate the equivalent diameter of a fictitious hollow, tor having the same equivalent inductance as the ong value of the inductance per conductor? 4.5. Find the geometric mean radius of a conductor an terms of the radius r of an individuat strand for (a) Three equal strands as snown in Figure 4.25(a) Four equal stancs as shown in Figure 4.25¢b) (a) (e) FIGURE 4.25 ‘Cross section ofthe srancec conductor for Problem 45 446. One cireurt ofa single-phase transmission line 1s composee af tree solid .5- ‘em radius wires. The return circust 1s composed of two solid 2 5-cm wires, The arrangement of conductors 1s as shown in Figure 4.26. Apy the concept of the GMD and GMA, find the inductance of the comple 1m millihenry per kilometer. 47. A three-phase, 60-Hz transposed figuration as shown in Figure 4.27 meter, The conductor geometric mean spacing Din meters is 2.9m, Determine the phase G@-p -G- op + mm spacing “The spac- ee per ld be the 138 4, TRANSMISSION Line PARAMETERS 49. A three-phase transposed line is composed of one ACSR, 47/7 Bobolink conductor per phase with flat horizontal ‘spaci Imas igure 4.30. The conductors have a diameter of 3.625 cm and a GMR of 1.439 cm. The line is to be replaced by & thiee-conductor bua. dle of ACSR, 477,000-cmil, 26/7 Hawk conductors having the same cross. sectional area of alumanom as the single-condtuct: 4 diameter of 2.1793 cr and a GiMR of 0.8839 em. The nes have a flat horizontal configurator 0 be operated ata higner voit age and therefore ine phase spac 14 mas measured from the center ofthe bundles as shown in Figure 4.31. The spacing between the conductors im the bundle is 45 cm, Determine (a) The percentage change in the inductance (b) The percentage change in the capacitance. a ° e OQ 021m dy - tim) Dy = 2m FIGURE 4.30 Conductor layout for Problem 49 (a. a > ‘ ° 5 ° a0 a10 a0 745 1 i — Dig = Wm a Day = 14m —t ———— Djs = 28 m —-____ FIGURE 4.31 Conductor yout for Problem 49 410. A single-cireutt three-phase transposed transmassicn lie is composed of four ACSR, 1,272,000-cmil conauctor per phase with nonzontal configuracion as shown in Figure 432. The bundle spacing 1s 45 em. The conductor code name is pheasant. In MATLAB, use command acsr te find the conductor di- ameter and its GAR. Determine the inductance and capacitance per pnase Ber kilometer of the line. Use function (GMD, GMRL, GMRC) =3ma (ATLAB to verify your result p— Di = m+ Dg = Lm, —Dy =28m FIGURE S32 het ayent tor Problem 4.10, 298909. Use function (GMD, GMI MATLAB to verify your re 1404, TRANSMISSION LINE PARAMETERS ‘29.coRoNA 141 a ca eae jee, en he FIGURE 434 Conductor fayout for Problem 4.12 t 4.13. A 60-H, single-phase power line and a telephone liN® are parallel io each ‘other as shown in Figure 435. The telephone line 18 symmetrically pos). tioned directly below pase 6. The power line eames afl ms current of 235 A. Assume zero current flows in the ungrounaed tephone wares, Find the ‘magnitude of the voitage per km incuced in the telephone line, @ 6 oe 3m Grn Figure 43s once yout for Problems 4.13 414. A three-phase, 60-Hz untransposed transmission line FUN5 tt parallel with ‘telephone linc for 20 km. The power line cames a balanced three-phase rms current of Jy = 32020" A, fy = 320/—120" A,and I, = 3202 —220° A. The line configuration 1s as shown in Figure 4.36. Assume 2070 current flows mn the ungrounded telephone wires. Fina ce magnitude of the voltage induced in the telephone line. (9) Bann nage 41s, Since easth is an equipotential plane, ine electric fu lines are forced the surface of the earth orthogonally. The earth effect ca” be Placing an oppositely charged conductor a deptn H belo the surface carn as shown in Figure 4.37(a). This configurasot WHhOUE the pre k CHAPTER > LINE MODEL AND PERFORMANCE 5.1 INTRODUCTION In Chapter 4 the per-phase parameters of transmission lines were abtaited. This chapter deals withthe representation and performance of transmission lines under ‘omal operating conditions. ‘transmission lines are represented by an equivalent ‘model with appropriate cwevit parameters on a “persphase” esis. The terminal voltages are expressed to neutral, the current for one pase and, ths the three-phase system 18 reduced to an equivalent single-phase system veloped for “short” and “medium the regulation and losses of lines and their opet erminal voltages are then considered. Next, long lice theory is presented and expressions for voltage and currere slong tne distributed line model ate obtained. Propagation constant and character istic impedance are definea, and itis demonstrated that the electrical pow: fransmitted over the lines at approximately the speed of light, Since the fermins conditions at the two ends of tne line are of primary importance, an equivalent a modal 1s abtamned by multipiyang lengtn 52 SHORTLINE MODEL 143, AZLAB functions are developed The phase tage at the sendiag end is fed at ene end of (3) A445, LINEMODEL AND PERFORMANCE {and since the shunt capacitance 1s neglected, the sending end and the receiving end current are equal, e., Is= IR 6a) ‘The transmission line may te represented by a two-port network as shown in Figure 5.2, and the above equations can be watten 1n terms of the generalized exrcust constants commonly known as the ABCD constants FIGURE 5.2 ‘TWwo-nor represemation ofa tanemussin tne, Vp = AVe + Bly Is = CVn + Dip (ie]=[2 2]f28] According to (5.3) and (5.4), for short line modet ‘or in matrix form, A=l BsZ C=0 Dai 6a Voltage regutation of the line may be defined as the percentage change in atthe recenving end of the line (expressed as percent of fll-oad vo fom no-load to full-oze. = Wares] ol iZ Percent VR = Wat i x 100 69 At no-load Ip = 0 and from (5.5) Vs Vaxwey = 2 5.10) fanny = = 10) For a short tine, A = 1 and Vayiwey = Vs. Voltage regulation is Jine voltage drop and depends on the load power factor. Voltage regu! 52 SHORT LINE MODEL 145 =6+j00 1465. LiNeMODEL AND PERFORMANCE The receiving end voltage per phase is 22020° V3 Vax = 12720 KV ‘The apparent power is Snes) = 381écos~ 0.8 = $8136.87" = 304.8 + 5228.6 MVA ‘The current per phase is given by Sh = 36.87" — Shige, _ 381L ~2687° X10" ayy _ ag gps ta Spe = ara = Moe ar? a From (5.3) the sending end voltage 1s Vg = Vig + Ziq = 12720" + (6 + 720}(10002 — 96.877 = 144.9324.99° kV 2 ) The sending end ye voltage magnitude is [Veqe—1)|= V3|V5| = 250 kV ‘The sending end power is Sao) = 3VSIE = 8 x 144.3344.03 x 1000£36.87° x 10-2 = 322.8 MW + 5288.6 Mvar = 433241.8° MVA Voltage regulations 250 - 220 Percent VR = 5 x 100 = 13.6% ‘Transmission line efficiency ts Pacaey _ 3048 = = SS x 100 = 94.4% 7 Boao) 7 305g 100 = OMA 381L36.87° x 10° 2 = irae = lonoeas.s7? a 43 MEDIUM LINE MODEL 147 ‘The sending end voltage 1s Ve= Vet Ziq = 12720" + ( ‘The sending end. [Mp cu] = Vs © kv ‘The sending end power is 1485. LINE MODEL. AND PERFORMANCE Ig Z=R+iX 1, Ip + | | + ve = yo a i viure sa Noma = oel fo du engi ine From KCL the current nthe series impedance designated by Ic. 1s = lat £¥n (19 From KVL the sending end vottage is Vs=Va+ Zl 18) Substituung for Ly, from (5.15), we obtain = (i+ 2) vet ttn 617 ‘The sending end curent is Ise t+ hve 518) Substitung for Tz, and Vs tov (+2) me (it 2) in 1519) Comparing (5.17) and (5.19) with (5.5) and (5.6), the ABCD constanss for the nominal model are given by a=(+Z) pez sam car(ie®) pels) a 3 In general, the ABCD constants are complex and since the m model isa sym ‘cal two-port network, = D. Furthermore, since we are dealing with « linet INE MODEL 149 lateral tworport network, te det Ys ey passive, (S38 ine oF the transmission matrix tn AD BC = 1 5.22) Solving (5.7), the receiving ence ext quuntties by [Yel e [ ite} ‘Two MATLAB functions Funcion (2, ¥, ABCD } essed in terms of the sending (5.23) tation of the transmission matrix, length are specified, and used when senes impedance wth are specified, The above functions provide options for the na the equivalent = mode! discussed in Section 5.1, Example 5.2 V. Uuree-phase cansimission n tong. The resistance per phase F = .036; g = 0, f= 60; L=0.8 ie ailia-Henry C= 0.0112; i, arero-Farea Lengtn = 130: = 335, YR = VR3pn/eqrei3) + +0; ¥ (receaving ena phase voltage) ¥, ABCD] = rledaccd(r, LC, %, t, Lengen) cos (0.8) . lrecerving ena pever) svang end current) ony ($8) = ABCD f column vector [Vs; Ia] YsIs(1) a = sqrt sending end L-L voltage) VsTs(2) cos(angie! 3e¥secon3( = (s3pn/acs she \ ‘sending ene current) (sending ena power factor) NVA (sendiag end pover) 39h *100 b> angie(ta)) aBcD(E,1)) = 7 150 5, LINE MODEL, AND PERFORMANCE s = Mg A’, Isn), gprinte( pe = he’. pfs) 3 = Ug LeL kV”, Ys3ph) s = Ng Mi, real(Ss)), Qs = kg Mvar’, imag(Ss)) fprant£(’ Percent voltage Reg = ‘hg’. RES) result in Eator 1 for Mediun line or 2 for long lane ~* 1 Moginal = moael 2= 4.68 + 3 39.2071 onns Y= 0 + 3 0.000548899 sienens 9.98924 + -j 0.0012844 4.68 6 + 9 39.207 trom | eeeacor SERS, boa £2 Blan | Is = 421.192 a pf = 0.869857 Vs = 345.002 L-t xy Ps = 218.851 MY Qo = 124.23 Mvar Percent voltage Reg. = 7.30913 Example 53 A MS-RV, three-phase transmission line 1s 130 km tong, The series impedance 2 = 0.036+ 0.3 0 per phase per km, and the shunt acmuttance is y = 34.22x10-* siemens pet phase per km. The sending end voltage 1s 345 kV, and the sending end current is 400 A at 0.95 power factor lagging. Use tne medium tine modet to find the voltage, current and power at ine receiving end anc the voltage regulation ‘The function (2, ¥, ABCD] = zy2abed(z, y, Length) is used to ootain the trans mission matrx of the line, The following cctamands 2 & .036 + 3* 0.3; y = j94.22/1000000; Lengtn = 130: Vs3pa = 345; Ism= 0.4; ka; 43 = -acos(0.95); Ys = Vs3pn/sqrt (3) + j+0; Is = Isme(cos(As) + j+aan(4s)) Y, ABCD] = zy2avcd(z, y, Longe); Vrlr = inv(aBCD)* (Vs; is}! Ve = VeIr(1); VeSpn = sqrt(3)*abe(¥r); % kV(recenving ena L-L voutage) tx = Vele(2); Im = 1000vabs(Ir): % A (receiving end current) pérs cos(angle(vr)~ angle(ir}); ‘h(receavang ena pover factor) St = 3Vreconj (Ir); MWA (receiving ene power? RY (sending end phase volcage! column vector (¥r; Tr] 151 spransé( Ye fpranté( Pr fprinti() Or = fpranti(? Perce: result 9 Enter 1 for Medium line or Nominal 7 30 Z= 4,68 + 5 29 ohms Y= 0*) 0.000546 siemens Pr = 224 Percent voltage aeg = 8.45: 54 LONG LINE MODEL For the short and medium ten: by assaming tne b and for 3 any pomt on the line are derved 1edel is obtained tor the Jong line 152 5, LINE MODEL AND PERFORMANCE Is T+ 42) zAr Iz) + + + Vs Vet As)~yAr yAz == V(2) | baz t F ‘ FIGURESS ‘Longline with distributed parameters. ‘Taking the limit as Az — 0, we have dv(z) az Also, from Kirchhoff’s current law =2I(c) Mn+ Aa) = I(x) +yArV(x + Az) Ma + Ox) ~ Ha) _ = =yV(et az} Taking te fimit as Oe — 0, we nave a) _ oy, a UVC Differentiating (5.26) an su evi) wat ‘The following second-order different £V (2) aa ituting from (5.29), we get _ l(a) dz zy V2} ~7V(z)=0 6.27 (530) 630 (532) ‘The solution of the above equation 1s V(x) = Aye Age (5.33) where 7, known as the propay ts complex expression given by (531) or ysatji= JW 534) "The real part ris knewn as the aver imaginary component 3 is known as the p length. =f fae ) 6.35) o Sect aire) 15.36) sven by Zo (5.37) Viz) = Ve, and Upon sut curent along (6.40) 1540 5. LINEMODEL AND PERFORMANCE ‘The equations for voltage and currents can be rearranged as follows: tee em ett V(e) = Vat 2.5 Ia (5.4) Lene ete ‘ a= pe 2 Me) = at te (5.42) Recognizing the hyperbolic functions sinh, and cosh, the above equations are want. ten as follows: V (a) = cost 72 Va + Zesimh-y2 Fp (5.43) 1) = eannraves cone ta ‘We are paricularty interested in the relation between the sending end and the re- cetving end ofthe line. Setung x = &, V(e) = Vz and (8) = 1, the rs V, = cosh 18 Vin-+ Zesinb of Ip (5.5) ie z sinh 1 Vp + cosh yf Ta 5.46) Rewnting the above equations in terms of the ABCD constants as bet. we nave [e]-[2 o]{%] A= cosh x Ba Zosianye (3.48) where C= fanmye Do coxhae 501 Now tat ate A= D aa AD ~ BC = 4 soy poses orate nde sa toreplace the ABCD constants of the wo-port network. Similar to the e (5.17) and (5.19) obtained for the nominal 7, for the equivalent « moxie! ven (14 22) em zt tear (12) ype (os 50) and (5.51) with (5.45) and (5.46), respectively, and making use Comparing of tne ident 4 _ coahyt=1 tanh 1 = coahato anny = nae the parameters of the equiv ane obtaneg, (8.53) (3.54) FIGURE 55 Egunalent = mode cr tong eng 3 f Length) and (2, ¥, lor the evaluation of Example $4 A 500-KV, three-prase transmy = = 0.045 ++ 30.4 9 per phase siemens per phase per km. Eval matox The following commands = sinalganma*Lenges. ABcd = fA Bc] 2-3) ¥ 9/20 © cann(gammatie ABCD = 0.9504 + 0.00551 10.8778 +98.36241 0.0000 + 0.00103 0.9504 + 0.00551. a= 10.8778 +98 36245 ye 9.0000 + 0.00104 5.5 VOLTAGE AND CURRENT WAVES ‘The mms expression for the phasor value of voltage at any point along given by (5.33). Sudstitvung a+ 58 for ‘Transforming from phasor domain to ume domain, the instantaneous voltage as a function of t and = becomes v(t } = VERA MIME 4. TR Ane, HA ¢5.53) lent wave. The second term decom lected wave, At any pointatong age 1s the sum of these two components. w(t.) = il, 2) + a(t, 2) (536 where vn(t,2) = VE Ave coslut +82) (sn valt.2) = V2 Age" costuit ~ Bz) (5.58) AS the current expression is similar to the voltage, the current can also be consid ‘red as the sum of incident and reflected current waves. Equations (5.57 ) and (5.58 ) benave like traveling waves as we move along This s similar tothe disturbance im the water at some sending point. To see consider the reflected wave va(t,2) and imagine that we ride along withthe wave, To observe the instantaneous value, for exampie the peak amplitude requires that IDCURRENT WAVES 187 ‘Thus, to keep up with tre wave and observe with the speea yea amplitude we must-travel (59) 6.60 The wavelengtn 2 of distance « on the wave ma phase shift of 2 radians Ban 8 (51) = 0, the real part of the constant becomes (5.62) Also, ine charactensite mpedance 1s pur aah (5.83) Ve tas the surge (5.60) the veioct length be- oo (5.64) Aw 5.65) ‘The expressions for the inductance pe © of a transmission line were denvea When the internal flux spacstance per unit length (4.58) ang (491), = GMRo. an (5.66) Gen Substituting for xo = 4r x 10-7 and eg = 8.85 x 10-", the velocity of the wave '8 obtained to be approximately 3 x 10* mee, ie. the veloaty of light. At 60 He, {he wavelength 1s 5000 km. Simnilarty, substituting For I and Cin (5.63), we nave 1 fim. GMD Fe FEV a GR, GMD 7 a 8 = 60In Fe (5.68) For typical transmission lines the surge impedance varies frem approximately 4000. for 69-KV lines down to around 250.0 for double circ For a lossless line y = 38 and tne hyperbolic functions cosh r=: 628 Bz and sinh yx = sinh j8 = j sm Bx, the equations for the rms current along the line, given by (5.43) and (5.44), become V(x) = cos 82 Va + jZ. sin Bz Ip (5.69) He) = IF onB2V_ +0582 In (3.70) Atthe sending end x = & Ves = cos BO Va + j2,sin BE In 670 Ts = p37 ein LV + cos BE Ie (5.22) For hand calculation it 1s easier to use (5.71) and (5.72); and for more accurate calculations (5.47) through (5.49) can be used in MATLAB. The termina! cond xe readily obtained from the above equations. For example, forthe open-cie Tine Jie = 0, and from (5.71) the nosloza receiving end voltage 1s At no-load, th 1 charging capacitive cus and the recet wsevideng from (5.73), as the line length anereases (¢ inereases art cos 38 ecreases, resulting in a higher no-toaa receiving end voltage. Fora solid short citout at une receiving end, Vig = 0 and (5.71) and (5.72) reduce to Vg = jZ.sin St Ip (5.74) Is = ces 68 Ip (515) ‘The above equations can be used (0 find the short cireuit curren at both ends of the line, URGE IMPEDANCE LOADING 159 5.6 SURGE IMPEDANCE LOADING 1 equal to its char- \y ces adc ing to the surge snown as the (SIE), given 50 MW for 230-kV sure of (ansmussion Fine We requirements are small AbU 5, LINE MODEL AND PERFURMANCE Example 5.5 A three-phase, 6-H, 500-KV transmission line is 300 km tance is 0.97 mH/m per phase and its capacitance is Os Assume a lossess line 15 km per phase. (@) Determine the fine phase constant @, the surge impedance Zz, velocity of prop ‘gation v and the line wavelength 2. (©) The receiving end rated load is 800 MW, 0.8 power factor tagging at 500 Kv. Determine the sending ena quantities and the voltage regulation, (a) For a fossless line, from (5.62) we have B = aVLO = 2n x 60VOST x COLTS x 10-* = 0.001259 radtkm and from (5.63), L z= 4% eo VG 0.97 x 10-> OST x 107, = 290.43 0 TTT x 10-8 = 79048 Velocity of propagation 1s Sse Seselap tor esey Neer rer iast Am 5 = gyl2gd x 10%) = 4900 (b) 3€ = 0.001259 x 300 = 0.3777 mad = 21.641" ‘The receiving end voltage per phase 1s 5000" Va= = 288.67520° KY aa eB The receiving end apparent power ts 800 got oe Suysey = Fog 4008" U8 = 1000236 87° = 800 + 7600 MA ‘The receiving end current per phase is given by SR 10004 — 36.87° x 10? = Shtas) _ 1000 = 115A TL - 3687" A Teo “aug ~ sx aesracoy — ~ 1517 — 38 1. The line induc 57. COMPLEX POWER OUGH TRANSMISSION LINES 161 1e sending end voltage 1s Vy = cos BE Vi + jZesin GE Ip = (0.9205)283.075.20° + 32: = 356.53216.5° kV £ ~ 36.87°)(1079) The sending end line-to-line voltage mo: al= From (5.72) the sending end curren Is = 17 sin Ol Ve + cost fa = 5pqq yg 0:9688)(288.67 = 00234-1709" A 1 ~36.87°) ‘the sending end powers x10? __ 386.53/0.9295 37 COMPLEX POWER FLOW THROUGH TRANSMISSION LINES form as A = |AlZ8,, B = |B|2@p, the sending end voltage as Vs = {Ws[28, and the recerving end voltage as reference Vg = |Vin|Z0, from (5.5) Ip can be wntten as, [Wolds ~1A}elVa) 20 Ine = (Blea (581) ‘The receiving end complex power is Srs0) = Prise +I Qac9) = Wal, (5.82) (5.83) (5.84) (5.85) 15.86) ‘The sending ena power Ssise = Psiae + Qsiag) = BLE (5.87) From (5.25), Is can be wntten 2s (5.88) Subst a Weve-ayll Vere 19 AF sett gg) ~ Wate=nlWaue di ZECTRANSMISSION CAPABILITY 163, on 6.91) (5.92) versus Prag) for fixed the recenving end power sstng the performance character ywreire( ABCD) is developed for m, and its use is demon sues of the trans tne construction of the receiving, sirated in Exarnpie For 2 lossless W°, and A = cos,4¢, and the s. To ascure an adequate 's usually Timited to 35 to ‘645. LINEMODEL AND PERFORMANCE ‘The expression for eal power transfer by (5.93). The theoretical maximum power transfer is when 6 = 90°. The pr operating ioad angle forthe line alone is limited to no more than 30 <0 45°. This 18 because of the generator and transformer reactanices which, when added to line, will result ina larger 6 fora given toad, For planning and other purposes, 113 press the power transfer formula in terms of SIC, até constnuct y curve. For a lossiess line X’ = Z_sin BE, and (5.93) may be Py= (See!) (Fees) ( ) sin 6 (596) anf ‘The first two terms within parenthesis are the per-unit voltages denoted by Vgp. and Virpuy and the third term is recognized as STE. Equation (5.96) may be wnitten as = MSmal Vaoul STL the line for a lossess line ts 5.9(i) shows that for short and medium ‘mum power transfer. Whereas, for longer loadability curve, As we see in the next section, for longer lines 1 to use series capacttors in order to increase the power transfer over the line. Example 5.6 ‘A three-phase power of 700-MW is to be transmitted to a substation km from the source of power. For a preliminary line design assum parameters: Vg = 1.0 per ani 6 = 30.87" (a) Based on the pre ime Joadability equation determine 2 nommal voltage evel for the transmission Tine, (®) For the transimission voltage level obtamned in (a) calculate the taeo ‘mum power that can be transferred by the transmission line. Vg = 0.9 per uns, A = 5000 km, Z_ = 3202. anc (a) From (5.61), the fine phase constant 1s 360 250 ans) = 29.68" Sp 25) = 22.58" From the practs Thus SIL = 499.83 MW From (3.78) = 400 KV is given by 2123.39 0 ted under steady state 8 (V5 = 10 166. 5, LINE MODEL. AND PERFORMANCE Consider a reactor of reactance X44, connected at the receiving end of 3 fong transmission line as shown in Figure 5.7, The receiving end current 1s (5.98) Note that Vs and Viz afe in phase, which is consistent with the fact that no reat ower is being transmutted over the line. Solving for Xteu yselds Kun = (5.99) For Vs = Va, the required inductor reactance 18 sin ae Xun = rc 15.100 1 cos 50” na mn between Is and I, we substitute for Vg from (5.98) into (5.72) Tg = (~F ont Xin + 08 08) 1 = (—Bam6tXn reso!) fa Substituting for X,,, ftom (5.100) for the case when Vg = Vq results 1 Is=-Ip (5.100 SP.LINECOMPENSATION 167 Wi ‘one reactor only at the receiving end, id the maxamum nse occurs will not be uniform, a0 exereise to show that 5 = Va, the voltage at (5.102) Also, the current at tne midspan ss zero, The fun opentine(ABCD) 1s used to find the cecewing end voltage of minated am an open circuit 1685, LINE MODEL. AND PERFORMANCE 5.9.2 SHUNT CAPACITOR COMPENSATION ‘Shunt capacitors are used for lagging power factor circuits created by heavy loads The effect is to supply the requisite reactive power fo matntan the recerving enc voltage at a satisfactory tevel. Capacitors are connected either directly to a bus bar (fo the tertiary winding of a main transformer and are disposed aiong the route to ‘minumize the losses and voltage drops. Given Vs and Vz, (5.85) anc (5.86) can be tused convenientty to compiite the required capacitor Mvar atthe recetving end tora specified leas. A function called stinteomp(ABCD) is developed for this purpose, and its use 1s demonstrated in Example 5.(. 5.9.3 SERIES CAPACITOR COMPENSATION ‘Senes capacitors are connected in sertes with the line, ustally located at th Point, and are used to reduce the series reactance between the load and the Point, This resusts in improved transient and steady-state stability, more cc ‘cal loading, and minimum voitage dip on toad buses, Senes capacitors have good characteristics that their reactive power production varies concurrently with ‘Swudies have shown that the addition of senes capacitors on EBV. 88 can more than double the transient stability load limu of long lines at a fraction of the cost of a new cransmission line, Is eee Ie ie 1 : ~iXcoer : | ie x, Mm ~iXesn Met, ‘s Ya a FIGURE 58 ‘Stunt ana senes eapactor compensation With the series capacitor switched on as shown in Figure 5.8, trom (5. ower transfer over tne line for @ ‘Where Xceer isthe senes capacitor reactance, The ratio X¢yep/-X" expressed as perwentage is usually referred to as the percentage compensation. The percentage compensation 1s in the range of 25 to 70 percent. 59.LINE COMPENSATION 169 One mayor drawback with senes 1s that special pro- bypass tie high current subsynebronous (5.104 1 equivalent tine 19 sags 3600 MVA +5 ebtamnea from (5.93) 1705, Line MODEL, AND PERFORMANCE Which results in 5 = 20.044°. Using the approximate relation given by (5.94), the Net reactive power at the recerving end is (500)(500) (500)? ° = £500)(500) 0) — OO osi21.641") = 28.15 Mae Qaise) = Tey £08(20.044°) ToFag C1641) = 23.15 Moa ‘Thus, the required capacitor Mvaris S¢ = j23.15 — j600 = —j576.35 ‘The capacitive reactance 1s given by 108 c= 2e[eOVaE aE) = OF The shunt compensation for the above transmission line including the fine resistance is obtained in Example 5.9(f) using the lineperf program. The exact so. ition resuits in 613.8 Mvar for capacitor reactive power as compared io $75.39 Mar obtained from the approximate formula for the lossless line, This represents approximately an error of 6 percent, (©) For 40 percent compensation, the series capacitor reactance per phase 1s Xen = O4X" = 0.4(107.1) = 4284 2 ‘The new equivalent circust parameters are given by Bs i(X" - Xi.) = (0 = 42.34) = 764.28 0 2 2 Y's = 3 tan(t/2) = 7-7 van(21.641°/2) = j0.001316 semens 17, SOnUBE/2) = aera tan(21.541°/2} = 0.00131 “The new B constants B = 964.26 and tne new A constant is given by Am ig 22 Ly, (90428)(70001316) _ 45 z 2 ‘The receiving end voltage per phase 1s 500 Va= 2 = 288.675 kV a= Fe = 288.675 — 1000/-36.87° BVg 3x WB ETSZOE In= = LIST -36.87° ka (E PERFORMANCE ROGRAM TL 565, ent VR = = 100 = 13% “The exact soluson obtained in Example by = SLO KV. Thit represents an error of 1,0 percent inalysis and compen. plays a menu with five models and the transmis following functions. sath) computes and returns the + rinchm, Lim mH, and C es and returns tne mode dance and admuitance pe iDabed\Z, V7 retums te ABCD constants when the + when t ince ond capacitance pe are specified. rameters per unit length an: 1725, LINEMODEL, AND PERFORMANCE. tons are defined in the MATLAB environment. Ifthe above functions are ed without the parenthesis and the arguments, the user will be prompted to enter required data, Next the Hineperf loads the program listmenu which displays 2 ¢ analysis and compensation. Selection of lowing functions. these options will call upon the givensr(ABCD) prompts the user to enter Vi, Pr and Qa. This f computes Vs, Ps, Qs, line losses, voltage regulation, and transmission effi givenss(ABCD) prompts the user to enter Vs, Ps and Qs. This function com: putes Viz, Pa, Qe, line losses, voltage regulation, and transraission efficiency, givenzi(ABCD) prompts the user to enter Vg and the toad impedance. This Function computes Vs, Ps, Qs, line fosses, voltage regulation, and transmission efliciency. openline(ABCD) prompts the user to enter Vs. This function computes Vi forthe open-ended line. Also, the reactance and the Mvar ofthe necessary react ing end voltage ata specified value ae obtarned, In 3 led at the receiving end ia order 1o maintain the specified Vz. Then, Vp 1 losses, voltage regulation, and transmission efficiency are found, sercomp(ABCD) prompts tne user o enter Va, Pr, Qn, power centage compensation (ie, Xe. of the specified series capacitor and Vs, P,Q: transmission effictency fer the compensate per .- « 100 ). This function computes the Mvar ine losses, voltage re: and ine, srshcomp(ABCD) prompts the user to enter Vs, Pr, Or, the desired the percentage sertes capacitor compensation. This function compute lend in order tance and the Mvar of a shunt capac fo mainian the specified Vp. transmussicn effictency are ol nT loads the pwreirel ABCD) w power factor, lage profiles for openvended 00 MW.08 ‘The eommanes ispiavs he tol Tht te tee MUL, ANY FERFURMANCE, Type of parameters for inpuy Select Parameters per unit legen (QD), g (siemens), L (ai), ¢ (uF) 1 Complex 2 and y per unit lengtn + y¥x (M), ¢ + 39 Csronens) 2 Nominal 7 or Eq. r model 2 A,B, 0, D constants 4 Coneuctor configuration and dimension 5 To quit ° Select musver of mena — i Enter line Lengta = 300 Enter frequency in Hz = 60 Enter line resistance/phase in Q/unit lengen, r = 0.016 Enter line inductance in mi per unit length, L = 9.97 Enter line capacitance an uf per unit lengua. © = _0116 Enver line conductance in siexens per unat length, ¢ = 0 Enter i for medtum lino or 2 for long line ~+ 2 Equivalent w noged 2S CSTE 5 107.419 cans ¥" = 8.9638e-07 + ) 0.00131631 siemens Zc = 290.496 + 5 ~3.35214 onns at = 0.00826172 nepex BE = 0.377825 radian = 21.6¢78" ‘Agcy = | 9-9295 + j000s0478 srl + 5107.12 /L-1Si4ie—06 + jo.onr2609 0.9205 + 50.0000078 | At this point the program listmenu 1 autom: lowing menu. Transmission line pertornance Analysis Sarees To calculate sending end quantities fer specified receiving end MW, Mvar {FORMANCE PROGRAM To caiculate receaving end qu for specified sending end MW, ! To calculate sending end quan tunen load impedance 1s speci! Open-end Ione and reactive conpensation Short-carcustes lane Capacatave conseasation Recesvang ena exrcle éiagran Loagabilaty curve and voltage profile To gust, Select number of senu — | Enver recervang end 3-pna: (G cor daggung ant ~ £2 ang ead quantities Lane perfor for spec vr = $00 RY Ps = 815.404 Wi, Us = PL = (8.4040 , QL = Percent Voltage Regulation = Transmission tine efficzency ~ ‘Memu) is displayed. At ine end of this analysis she Fistmenu (A. rmance when 600 MW ‘Selecting option 2 of the listmenu results in Enter sending ena line-line voltage kV = 525 Enter sending end voltage phase angle’ = 0 Enter sending ena 3-puase power HW = 660 Enter sonding enq 3-phase reactive pover (+ for lagging and ~ for leading power factor) var = 460 Laue performance for specitiea sending end guaitities Vs = 625 kV (L-L) at 0° Ps = 600 MW, Qs = 400 Mvar Is * 793.016 A av ~33.6901°, PFs = 0.89205 lagezag Vr = 417.954 kV (L-L) at -16.30¢4° Is = 1002.6 & at -52.16° PFr = 0.810496 lagging Pr 588.261 MY, Qr = 425.136 Hvar PL = 11.7390 HW, QL = 25.136 Hvar Percent Voltage Regulation = 35.1303 Transmission line efficaency = 98.0435 (€) Determine the sending end quantities and the line performance When the ce cewving end load impedance 1s 250 at 500 kV. Selecting option 3 of the listmeny results in Enter xeceivirg enc Line-line voltage kV = 500 Enter recervang enc voltage phase angle” = 0 Enter sending ena conplex load ampecance 200+) *0 Lane performance for specified load impedance We = 800 RY (L-L) az 0° Tr = 995.431 A at 0° PFr = i Pr = 862.069 Mi, Qe = 0 Mvar Vs = 507.996 kV (L-L) at 21.8037° Is = 995.995 A at 21.7842", PFs = 0.999988 Leading Ps = 876.341 Mi Qa = -4.290 Hvar PL = 14,272 Mw QL = ~4.290 Mvar Percent Voltage Regulation = 9.30464 ‘Transmission line efficiency = 98.3714 (G) Find the recewing end voltage when the line 1s terminated in an open cxrcutt, and is energrzed with 500 kV at te sending end. Also, determine te reactance and the Myar of a th the ns Eator sonding en¢ Lino-line volt: Enter sending ena voltage phase angie" = 0 Open Line and siuunt reactor como: 0020284 tage = $00 Shunt reactor re Shunt reactor r Voltage profile of aa wnt 1785, LINE MODEL AND PERFORMANCE. Selecting option 5 of the listmenu results im Enter sending end Line-Line voltage kV « 500 Enter sending ena voltage phase angle” = 0 Line short-carcunted at the receaving end Ys = 600 kV (L+L) at 0” Ie = 2692.45 A at 87.5549" Ts = 2502.65 A at ~87.387° the rect line is energized sng end to Keep the receaving end voltage at $00 ‘500 kV at ane sending ena. Selecting option 6 will display the compmenu as follows: Capacitive coapensation analysis Sexect Shunt capacitave compensation i Series capacative compensation 2 Serses ana snunt capacatave compensation 2 Te quit ° Seiecting opuon 1 of the compmenu resilis in Enter sending end line-line voltage kV = 500 Enter desired receiving end line-line voltage kV = 300 Enter receiving ena voltage phase angle” = 0 Enter receivang ena 3-pnase poxer MW = 800 Enter receivang end 3-puase reactive power (+ for lagging ana - for leading power factor) Nvar = 600 Ys = 500 BY at 20.2479" vr = 500 kv ato Pload = 600 MW, Qhoad = 609 Load current = 1184.7 4 at ~26 Requized shunt capacitor: 407 Shunt capacitor current Pr = 800.000 vi, Is = 940,308 Ps = 812.489 PL = 12.469 MK Percent Voltage Reguiaticn = Transmission Jane efficiency ~ Enter percent compens: Reccsmendea range 28 t Series capacitor compensatzex LANCE PROGRAM 179 wpensated by senes at $00 KV. 2.00 Ns PRAPURMANCE PROGRAM. TST AOU 2. LINE MULL ANU FEREURMANCE t Ve1013V, (3) The line has 40 percent series capacitor compensation and supplies the load in (a). Determine the Mvar and the capacttance of the shunt capactors to be installed at the receiving end to keep the receiving end voltage at 500 kV when line 1s energized with 500 kV at the sending end, i Selecting option 3 of the compmenu resu Enter sending end line-lane voltage kV = 500 Enter desired receiving end Line-line voltage kY = 500 | Enter receiving end voltage pasae angle” = 0 i Enter receiving end 3-puase power ¥W = A00 Enter recezming end 3-pnase reactive pover (+ for lagging and ~ for leading power factor) var = 600 Enter perceat compensation for seraes capacitor (Reconmendea range 25 to 75h of tne line reactance) = £0 | Series and shunt capacator compensation Series and shunt capacator compensation Vs = S00 KY (L-L) at 12,0224° Vr = 500 KV (t-L) ar of i Ploaa = 800 MW, Gloag ~ 600 Kvar Goad current = 1154.7 A at -36.8699°, PF = 0.9 iagging Required shunt capacator: 432.736 2, 6.1208 uF, 577-72 iver Shunt capacitor curreat = 567.093 A at 90" Required series capacitor: 42.8476 9, 61.9074 uF 27.7274 Mar Subsyncnronous resonant frequency = 37.9473 fiz Pr = 800 MW, Gr = 22.2804 Hvar Te = 924.119 A at -1.5959°, PFr = 0.999612 lagging Is = 961,165 A at 21.5977", PFs = 0.966068 leading Ps = 812.257 MY, Gs = -137.023 tvar PL = 12.257 MW, QL = ~189.304 Mvar Percont Voltage Regulation = 4.41619 Transmission line efficiency = 98.491 (2) Construct the receiving end ctrcle diagram. Setecting option 7 ofthe listmenu results n Yer mn i jeleage profile curves u i me loacaoslaty curs 2 Enter veceavang end line-Line voltage kv = 500 | fees 9 | A plot of the recerving end exrele diagram is obtained as shown m Figure §.10 Selecting opt the profimenu re 182 5. LINE MODEL, AND PERFORMANCE ‘Voltage profile for engtn up to 1/8 wavelength, Z = 290.5 ohms Rated loa | Shrt-ckt 0 ~ 100200300400 S0—«GCG TOD Sno Sending end FIGURE 511 Voltage profile for length up PU, SL A wavelenst, Loadabi SIL 860.8 MW, cetta = 30 degrees ‘Theoreucat stabi ractical line Toad 030040060000 F0H0 Line lengtn curve for length up io 1/4 wavetenst, eng end, curve for tength up to 1/4 wavelength 1200 INE PERFORMANCE PROGRAM 183 Enter sending end line- Enter rated sending ena Enter power factor = 0.8 foizage XV = 500 voltage XV = 500 = 500 aty, Amp/phase = 3500 Envar Enter Enter The line loads sv an Figore 5.12 (page 182) PROBLEMS 5a. A) ss 16 kin long, The line nas a per per kin. Determine the sending. power, and the transmission G34 x 10°° siemens pe mod (a) The tans 2 onts. 5, LINE MODEL AND PERFORMANCE Find the sending end voltage and current, voltage regulation, the. ‘poWer and ine transmussion efficiency when the line delivers (b) 200 MVA, 0.8 lagging power factor at 220 KV. (©) 305 MW, unity power factor at 220 KV. Use lineperf program to verify your res -phase transmission line 1. Shunt capacitors ae installed atthe receiving end to improve the line peor | mance of Problem 5.3. The fine delivers 200 MVA, 0.8 lagging power factor at 220 RV. ine efficiency i 5.3. Obtan the si @ in the total Mvar and the capacitance per phase of tne ¥-conaect Leese i : oe gram. Then, s Cspactos nen the sending end voltage 16 200 YH: Woe (235 (3.86) compute th power sng d andthe recomng enteaciveporet | (b) Use lineperf to obtain tne compensated line performance. | Fee-phasc, 345-kV, 60-Hz transposed line 1s composed oi wo ACSR. ,000-cmil, 45/7 Bluejay conductors per phase with flat norizor ing of 11 m. The conductors have a diameter of 3.195 em and a (268 om. The bundle spacing is 45 cm. The resistance of exch conductor 1m the bundle 1s 0.0538 © per km and the line conductance 18 0% ‘The line's 150 km iong. Using the nom: (ey Determine A= D = 0.98182 + j0.0012447 B= 4.035 + j58.947 © = j0.00061137 | 7. Write a MATLAB function named [ABCD] = abedm(z, y, Ln: and retum the ABCD transmission matrix for a medium sion line where 2 is the per phase series impedance per unit feng stunt admuttance per umt length, and Lngt rs the line length. Then, wate & program that uses the above function and computes the recesving end quan ‘ities, voltage regulation, ang the line efficency when sending end qu are specified. The program should prompt for the following quantities ‘The sending end The sending ena v ine voltage magnitude in kV “ 3 Ir logging power age phase angie 1n degrees : 186.5. LINE MODEL AND PERFORMANCE factor, Detetmme the total Mvar and the capacitance per phsse of the Y- ‘connected capacitors to keep the receiving end voltage at 735 KV wnen sending end voltage is 765 kV. Hir power angle & and the Quantities and voltage reg 5.12. Senes capacitors are installed at the midp and the voltage regulation when power factor at 735 kV. 5.13, Senes capacitors are installed atthe midpoint ofthe line in Problero 5.8, pro- viding 40 percent compensation. In addition, shunt capacitors are mstalled at the receiving end. The line celivers 2000 MVA, 0.8 lagging power factor De termine the total Mvar and the capacitance per pase of the series a ‘capacitors to keep the receiving end voltage at 735 KV when the sen voltage 1s 765 kV. Find the sending end quantities and vollage regu tne compensated line. Problem 5.8 has a per pnase re present a surnmary of the ion along wi ommendations. (@) Determine the sending eng quantities for te specified recerng end ‘quantities of 79520", 1600 MW, 1200 Mar. (6) Determine the receiving end quanuties for the specified sendiny end ‘quantities of 7650", 1920 MW, 600 Mvar. ©) Determine the sending end quantities for a load impedance of 282.33 + 02a 735 KV, id the receiving end x ge when the line is term KV at ine sending end, a three-phase shunt reactor to ing end voltage to 7 ted and the compensate fed at the recerving end to keep che fend voltage at 735 KV when line is energized with 765 KY. Obtaia the li performance of the compensated line (@) Determine the line performance ‘wien the line 1s compensated by sees capacitor for 40 percent compensation with the load condition in past (3} at 735 kV, INE PERFORMANCE PROGRAM 187 pensation and supplies tne ictance of the shunt capac- receiving end voltage at Wg end voitage 1s 765, sn-carrying capacity a tine are Obtain the sending end quantines a regulation when line 1000 MVA at 0.8 lagging powe: ww. at both ends: iapensated 1g and may sending end. When the load at end is 700 ‘nd the past wmpedance Ze. end to keep [Vs] = i rewctantee per phase From a preliminary tine esign, the ine phase constant and surge impedance are given by J = 9.49 HH 10~ radianvhan and Z, = 343 0, respective, Based on the practic! line loadebility entena determine the suitable nominal, CHAPTER Voltage tevel in kV for each transmission line. Assume Vj = 1.0 per Va = 0.9 per unit, and the power angle 6 = 36.87° “18. Power system studies on an existing systom have indicated that 2400 MW smited for a distance of 400 km. The voltage levels being Fonsidered inchude 345 KV, $00 KV, and 765 KY. For a preliminary design ‘you may assume the following surge POWER FLOW ANALYSIS B45kV Ze = 9209 SO0kV Ze = 2900 T85KVY Zo = 265.2 ‘wavelength may be assumed to be 5000 km, The practical line load Ability may be based on a load angle J of 35°. Assume [Vs} = 1.0 pu and {Wel = 0.9 pu, Determine the number of three-phase transmission required for each vottage level. Each transmassion tower may nave up to two the corona loss, all SO0-KV lines must have at least two con, per phase, and INTRODUCTION MYA. Use acst command in MATLAB to find a surtable conductor sive Following are the minimam recommended spacings between adjacent phace ‘conductors at various voltage Levels the previous chapters, mous (@) Select a suitable voltage level, and conductor size, and tows Use lineperf program and option 1 to obtain he voltage rea {ission efficiency based on arecetving end power of 3000 MVA at .$ p. factor lagging atthe selectea rated voltage. Modify your cesign and s. \uctor size for a line efficiency of at teast 94 percent forthe above spec. ied load, (b) Obtain the line performance ineluding options 4-8 of the ‘gram for your final selection. Summarize the line characte iced line compensation, 189 1906 POWERFLOW ANALYSIS: In this chapter, the bus admittance matrix of the node-voltage formulated, and a MATLAB function named ybus is developed for the sys atic formation of the bus admittance matrix. Next, two commonly used tecluniques, namely Gauss-Seidel and Newton-Raphson metiods for tne of nonlinear algebraie equations, are discussed, These techniques are empl i pr pnewton, and de~ by Gauss-Seidel, Newton-Raphson, and the fast decoupled power flow, respectively 6.2 BUS ADMITTANCE MATRIX In order to obtain the node-voltage equations, consider the simple power sy shown in Figure 6. where impedances are expressed 1n per unit on 2 MYA base and for simplicity resistances are neglected. Since the nodat based upon Kirehiofi’s current law, impedances are converted to ada i 1 wots By Ty FI8y FIGURE 6.1 ‘The impedance diagram ofa sumpie system, ADNOTANCE MATRIX. 191 Node O.9 sndeends Rearranging these equations ¥ heat We introduce Yas = yis +929 + oa Yau = vse Yin = Ya = -y2 Mis = Yar = -s You = Yor = —a You = Yas = —yoa ‘The node equation reduces to 1 =Yuvit Viola + Yaa + Yas tn = Yar Vi + YoaVe + YosVa + YaaVe In = Yuna + YaaVa + YaaVa + YuaVa N= Yai + Yaa t Yaa + Yuve In che above network, since there is no connection between ous 1 and 4, Fig = Yq. = 0; sumifarty Yo) = Yan = 0. Extending the above refatton to an n bus system, the qode-voltage equ: i matrix form 1s h Ya Yin Ya Yin | TM bh Yar Ya Ys > Yan | | Va bi* Ya X v oe : In Yau Yer ~~ Yor 0+ Yon | | Va or Tus = Yue Vins (62) where Thus 1s the vector of the injected bus currents (i.e. extemat current sources) ‘The current is positive waren lowing cowards the ous, and its negative if owing away from the bus. Vous 1 the vector of bus voltages measured fram «ne reference Node (i.e, node vottages), Yiu is Known as the bus admutance matrix. The diag ‘onat eicment of each node 1s the sum Of admittances connected toi It is known as the self admittance or driving point admtiance, Le, =D get 63) i} ‘The off-diagonal element 13 equal to the negative of she admuttance betweer nodes, Its known as the mutual admattance or transfer admuttance, 1.2, Yo = Yu ty a) 2 BUS ADMITTANCE MATRIX. 193 ‘When the bus currents are known, (6.2) can be sol ‘or the m bus voltages. Vius = Yin Tour 65) ‘The mverse of the bus acinitiance matnx 15 knawn as the by 2oae. The adnittance matexx obtained with one gular, Otherwise the nodal mart Inspecivon of the bus admittance mate along the leading diagonal, and we need to st tance matrix only. Ina typical pawer syseem ute 15 symmetric ipper trangutar nodal ad- pute ats inverse, By mes the mverse of a sparse tors, thereby gn of an apropnatcly XK can be expressed as an advantage in computat cound-off errors. However, Zou, Which 18 rey obtained df ‘aversion, This tectimique 1s discussed 19 Cha Based on (6.3) and (5.4), nained . storage and reduction of anaiys: j-8e 0 called Y= ybustzdatay Admittance matnx. adata 1s the first ewo columns are the line bus line resisiance ana reac matrx, The 4 nning colvmns contain the fetus the tus admittance mnderstand ene 100, frst converted to admutionces. ¥ 15 line tata is searched, and che off-diagonal elemne: ‘oop, line cata 18 searches to find the elements co efements are thus fo im a nested and the diagonal “ The foliowing 1s a program for be bus a 2 mata funccion{t} = yous(zaatal zdeta(.,1), averdatal 2) Xezdatal.,4); norelengta(caata(.,i)): nous 2), aex(nz)) 14 6, POWER FLOW ANALYSIS yo ones (nor, 1). / Yoranch aanatcance Y = zeros (nbus .nbu: 4 imtialize ¥ to zero % formation of the off diagonal elements 42 ml) > 0 & ar(k) > 0 (@) = YQLGO ,zr(k)) = y0); 2) = YGn1 (4) jnx Cie) for a= t:npus 4% formation of the diagonal elements for k= tinbr if al(e) <= n | arte) == a Yan) = Y(ayn) + ¥G0 else, ena end, end, Example 6.1 ‘The emfs shown in Figure 6,t are Bi = 110° and By = 1.00% Use the nce tton Y = ybus(adata) fo obtain the bus admittance mataix. Find the bus impecance ‘atnx by taversion, and solve forthe bus voltages ‘With source transformation, the equivalent current sources are The following commands % From To z= (0 Y = yous(a) Tous = (-J41.i; -3#1.28: 0; 0]; Zbus = anv(¥) Vous = Zbaseibus i bus admittance matrix % vector of bus currents 4% bus ampedance metrix result in 63. SOLUTION OF NON 63 SOLUTION OF NONLINEAR ALGEBRAIC EQUATIONS 0 + 0.5453, 0 + 0.6254, very inefficient, It ents Zbus = The above function is rearranged and written as 2=9(z) on is an initial estimate of the vanable =, the following sterattve sequence 1s 20) og 63) A solution 1s obtained when the difference between the absolute value of the suc- cessive iteration is less than a specified accuracy, ie, Where ¢ is the desired accuracy. + 2 ce 69) Example 6.2 ‘Use the Gauss-Seidet method to find a root ofthe following equation (2) = 29 ~ 62? +92-4=0 Solving for 2, the above expression is watten as ‘The MATLAB plot command 1s used to pi as stiown in ‘peated root at x = 1, Apply the Gauss-Seidel algorithm, and use an estimate of 2a From (68), the fist teration is Hatta tiiisalb eatin 02) = Hoy + Bo? + $ 22m ‘The second iteration 8 2) me o(2.2202) = 2 (2.2092) + St222a9y? +$=25173 ‘The subsequent steratons res 4.0000. The proces: In 2,866, 3.3376, 3.7398, 3.9568, 3.9988 ana repeated until the change in variable 1s within the desiced. iE | The resutt ss 63.SOLUTION OF NONLIVEA® ALCEBRAICEQUATIONS 197 5 set to a hig value Initial estimate Iteration counter ading for results t for convergence of aterations an variable ssave approximataon 198 5, POWERFLOW ANALYSIS. Iter g ox x i 2.2292 0.2222 2.2999 2.5173. 0.2081 2.8173 2.8966 0.5793 2.8966 3.3376 0.4410 3.3376 3.7398 0.4022 3.7398 3.9568 0.2170 3.9868 3.9988 0.0420 3.9988 4.0000 0.0012 4.0600 4.0000 0.0000 4.0000 Osea) yo eervonren 1m decomes aD 2 9 + al (6.10) Example 6.3 ywek + Find a coot of me equation in Example 6.2, using the Gauss-Seidel method sith an acceleration factor of a = 1.25: Starting with an inital estimate of 20° = 2 and using (6.10), the frst eration 8 Lion? 4 Sint a 4 = ~2(2)° + S09)? + 4 = 2.2022 gh + ge +5 = 2+ 1,252,222 ~ 2) = 2.2778 ‘he second iterauon s (22778) = —Z2.2778) + Sc2.2r7e) + 4 = 28002 2 = 2.9778 + 1.25)2.5002 — 2.2778) = 2.6683, The subsequent serations reslt in 9.0801, 3.1881, 3.7238, 4.0084, 3.9973 ana 4.0005. The effect of acceleration is shown graphically in Figure 6.4. Care must be taken an dhgigesce, in the MATEAB commana of Example 6, repince tne command before the end sateen by 2 = = 125 saz toreect tc eff ofthe acceleratoa factor anc maf te progran. eearranged 2006, POWER FLOW ANALYSIS 1 in the variables ace within the specified accuracy, a solution fas converged, ott- erwise another -‘sration must be performed, The rate of convergence can often be (6.15) increased by using a suitable acceleration factor ce, and the tterative sequence be: comes (6.16) 26) = al + a(aftg — 21") «6.13) 6.3.2. NEWTON-RAPHSON METHOD (6.17) ‘The most widety used method for solvin 16.16) can be ceasranged as tion procedure based on an in senes expansion. Consider the s of the one-dimensional equation given by f(z) =e (6.1) ate of the solution, and A:x(°) is a small de 1 We must have f+ do) =e Expanding the left-hand side of the above equation in Taylor's senes abo yrelds @ sal (£)” 220 3 (2) at nee a, 2 \ aa? ‘Assuming the error Az(! is very small, the higher-order terms can be neglected whien rests in Ac = (2° Az «= where Ac? =e Fa) Adding Ax! to the initial estamate will result n the second approxi 6) — 4 = 30 \ 202 6, POWERFLOW ANALYSIS: 30 i(z)= 2 - 627 492-4 20) 10) 0 of ——+ Sear ao ricune ss ‘Grophicelillustation of he Newton-Raskson algorithm, (*)° = 3(6)" 126) +9-= 45 ag, “The subsequent iterations result in oun a2 4 Ant 2 4.9889 - ee 42789 Sete 2.9981 : 2a) 4 aa 24279 = 40405 = 2 4 As = 4are9 - 2 ores 1 Ach? = 4.0005 - SG = 4001 2) oo alg Aol) wo 4.pons — 22095 _ 4.0000 9.0196 We see that Newton's method converges c Seidet metnod, The mi or diverge if me stamag may converge by Ute Newtor-Rapason method. Bec? + Sex +9 3 63 SOLUTION CF Ko lue 1s not clase eno ‘The following commands show the procedue oxet; ‘ange an varinn.e primput (Eater initial estimate - 1 ater 3 0; disp(at De J hile aps(ax) >= 0.001 & xter « er), disa([De, oy AATEDRAIC EQUATIONS 203, ly than the Gauss. from the expected one ron of the given equation eee a pigh value Tatial estimate ~ { Toeration counter ~~ 2°) % Weading for convergence No. or xterations w Reszawal te? Derivative change an variaple ‘cessive solution 204 6. POWERTLOW ANALYSIS; ‘or in matrix form, on = (fay In short form, 1tcan be wniten as Act) = Max Ax =], (6.19 and the Newton-Raphson algorithm for tne n-dimensional case becomes xtra (6.20) where az! 62) 6.22) (36) Gy ge)" J is called the Jacobian mamrx. Elements of this matrix are the paral cess te problem to solving a set Improve the accuracy of the estimates. The solution of (6.19) by mversion is very inefficient. IC 1s not ned to obtain the inverse of J), Instead, a direct solution 1s obtained by opt ordered trianguiar factonzation. In MATLAB, the solution of linear equations AC’ = JAXX is obtained by using the maunx division opera AX = J\ AC) which is based on the tnangular factonzation and Gaus inanon, Example 6.5 1¢ Newton-Raphison method to find the ant 63. SOLUTION OF NONLINEAR AIC EQUATIONS 205 tuons of the curves n the following xeinput(’Enter initial estimates, col. vector{xt;x2]>°) Dx = (1; 1]; % Change ia variable 1s set to a nagn value cota; ah; disp( Iter 0¢ —Jacopaan matrix Dx x) 4% Heading for results while max(aps(Dx)) >= 0.0001 & iter <10 Convergence tact itersaterti; 4% Ieeration counter £ = De(t)724x(2)°2; expCeCt))+x(2)]; % Functions De=c~ #; 4% Resaguals Js (2G) axa) % Jacobian natrax exp(x(1)) 0 De=J\De, 4 Change in varsaples xextDt; 4% Successive solutions fpranti('%g’, iter), disp( (De, J, Dx, J) % Resuite: ena, When the program 1s run, the user is prompted ¢o enter the intial estimate ‘Let us try an initial estimate given by (0.5; Enter Initaal estimates, col, veetor (x, 22) + (0.5; —i1 Tter AC Jacopian matrix 2 i 2.7500 1.0000 -2.0000 1.3034 0.3513 1.6887 1.0000" -1.9733 2 1.8928 2.6088 3.9066 isoa73 0.7085 3.6818 i.0000 1.7389 ri 3 0.1205 2.0948 -3.4778 1.005; sd 79.1111 2.8499 i.co00 <17296 = 4 -0.0019 2.0102 ~3.4503 1.0042 0.0025 2.7321 i. 0c00 71.7296 5 -0.0000 2.0083 -3.4593 1.0082 0.0000 7.7238 i 0600 -i-7298 Alter five iterations, the solution converges 102; = 1.0042 and x = accurate (© four decimal places. Starung with an initial value of [--0.5; 1s closer (o the other intersection, resuits 11 5, = —1.8163 and ty = 0.837 Example 6.6 = 1.2) = 1, and-ay = 1, solve te following y the Newton-Rapnson metnod oslad =u nat 3} - S29 3 nhnims = 6 wou ns 207 Taking partial 7 convergence of iterations % Functions 1% Rosaduats ian aatrax rn varzable 64 POWER FLOW SOLUTION control of an existing system as well as planting its future expanston. The problem consists of determining the magnitudes and phase angle of voltages at ach tus and active and reactive power flow in each line. In solving a power flow problem, the system is assumed to be operating uncer balanced conditions and a single-phase model 1s used. Four qui ated with each bus. These are voltage magminude |V/ and reactive power Q, The system buses are generally classified into three types. Slack bus One bus, known as slack or swing magnitude and phase angle of the vol the difference between the scheduled lozds and generated power ‘caused by the losses 1n the network, is taken as reference where the Regutated buses These buses are the generaior buses. They an voluage-controlled buses. At these buses, the real power and voltage i: tude are specified. The phase angles of the voltages and the reactive power are to be determined. The limits on the value of the reactive power are specified. These buses are called P-V buses. 64.1 POWER FLOW EQUATION Consider a typical bus of & power system network as shown in Fi mission lines are represented by there equivalent 7 models wha ‘been converted to per unit admuttances on a common MVA base nevi D V5 ey ‘The real and reacnve power at bus its PQs = ME (6.25) 16.26) 620 power flow ‘must be solved (6.28) where yy shown in lowercase letters is the actual admittance in per unit. Pr and (f° are the net real and reactive powers expressed in per unit In weitag the KCL, current entering tus i was assumed positive, Thus, for buses reactive powers are injected into the bus, such as goncrat hhave positive values. For load buses where real and reactive powers are flowing away fom the bus, P2* and Q7* have negative values. If (6.27) is solve and Qi, we have 247 620 ‘The power flow equation is usually expressed in terms of tne elements of the bus admittance matnx. Since the off-diagonal elements of the bus adr matrix jus, shown by uppercase Is are Vig = 3 yj (6.28) becomes ance ae Yij = yyy, and the diagonal clements an and PHD Lary a8 (632) Qi) 2-2 aes (6.33) Yi nctudes the admuttance fo ground of line charging susceptance and any other fized admuttance to ground. In Section 6.7, a model is presented for tra containing off-nominal ratio, which includes the effect of transformer tap & ge are specified for the slack bus, th 2(n ~ 1) equations which must be solved by an iterative method, Under Power clemand, whereas the scheduled voltage at the generator buses are somewhat hhigner, Also, the phase angle of the load buses are below te reference angle in accordance to the rea) power demand, whereas the phase angle of the se ator 65. GAUS5 1. FOWERFLOW SOLUTION 211 buses mnay be above the reference value ¢ fowmng into the bus. Thus, of 1.0 + 70.0 for onknos correlates with the actual pet pe amount of real power However, since | cad its reat partis selected in e pone specified accuracy, 1.2 For the power mis rance must be 6.6 LINE FLOWS AND LOSSES After the iterative solution of bus voltages, the next step isthe computation of line flows and line tosses, Consider the line connecting the two buses 1 and j in Figure 6.8. The line current Fj, measured at bus 4 and defined positive :n the direction Lo Tyo] vo ¥50) FIGURE 63 “Transmssion line model for ealeussting line ows. 1 => jis given by Fg = Tet fo = vis(Vi ~ Yj) # yo Similarly te ine curent J; measured at bus j and defined postive in te as J fis given by Tis Tet Tyo = vg Vj ~ Vi) + ys (6.39) ‘The complex powers S,; from bus sto 9 and Sy from bus ) to sare Siy = Villy (6.40) Sue = Vili 64H The power loss in ie 1 — J 18 the algebraic sum of the power flows determined from (6.40) and (6. Suig = Sy + S55 (6.42) ‘The power flow so following two examples. n by the Gauss-Seidel method is demons: ‘Example 6.7 Figure 6.9 shows tne one-line diagram of a simple thee-ous power system with generation at tus I. The magnitce of voltage at bus 1 1s adjusted to 1.05 per OWSAND LOSSES 213, ed on the diagram, Line ine charging sus- impedances ave marked in per ceeptances are negiected. 1 2 + 70.04 i omarion ca [7 Mw i O.01 + 50.03 we soc | t 1386 45.2 MW Mar the voltage at \gram show- Similan, marked on the etwork shi Atthe P-Q bases, the complex loads expr (236.6 + 9110.2) pu for hand caleu- ng from Vy ana Vy ace ‘AA 6, POWER FLOW ANALYSIS 7 ya = 10-20 H |_.. 256.6 O 7 tng = 10 ~ 20 ma=i3—jo | 102 Slack Bus 3 [ Vy = 1.050" | t 185 452 MW Moar FIGURE 630 ‘One-line diagram of Bxampie 6.7 (aamnttances m pu on 160-MVA base) SBS NOE 4. (10 — 520)(1.08 + 70) + (16 ~ 532)(1.0 + j0) (8 = 382) = 0.9825 ~ 70.0310 and oy _ RE tari spar ya oe 7 Ve + ix _ THESES OME? + (19 — 790)(1.05 + 30) + (16 — 732)(0.9825 — 30.0310) = (26 ~ 76: = 20011 ~ 0.0989, Me fe i |, For the second iteration we have wt v SRSA + (10 - 520)(1.00 + 40) + (16 — 332)(1.00:7 — 30.0383) a sense joanns + (10 = SPOR OO + 70) $6 © 382M 10081 = 70.0883), Na (26 — 782) = 0.9816 ~ 30.0520 and, 2) _ SOARES + (10 - 790)(1.05 +70) + (8 - 332)(0.9816 — 50.052) Pes (26 = 762) = 1.0008 — 70.0459 ‘The process 1s continted and a solution 1s converged with an accurecy of 5 x 10-? per unit in seven iterations as given below. vi) = 0.9808 — 50.0878 ve = 1.0004 - 70.0488 PLOWS ANDLOSSES 215 = 0.9803 — 70.0594 = 0.9801 - 30.0598 = 0.9801 — 70.0599 = 0.9800 ~ 70.0800 vs" 0002 ~ 30.0497 1.0001 — 30.0499 10 — j.0500 ‘The final solution 1s Vp += 0.9800 ~ 70,0600 = V5 = 4.0000 — 70.0500 = 5 MW and: With line charging = 19-j08 20-910 eh) = (1 = J0.08)] = 64+ 748 : wl ef gg = 1995+ j084 pu ows aNDLC 2 216 6. POWER FLOW ANALYSIS YE FLOWS ANDLOSSES 217 ‘Sox = Val = (1.0 — j0.05)(—2.0 — j1.0) = ~2.05— j0.90 pu 0.02 + 30.08 . = ~205.0, MW — 590.0 Mvar EEE fe 400 Sas = Valfjy = (0.98 ~ 50.06)(~0.656 + j0.48) = —0.656 — 70.432 pu a oo = -65.6 MW ~ j43.2 Mvar Co a Sea = aly = (1.0 ~ 30.08)(0.64 + 70.48) = 0.664 + 50.448 po G.01 + 50.08 0.0128 + 30.025 [> ia = 65.4 MW 4 jdd.8 Mvar Slack Bus ee Ve = 1.050" 4 I and the line losses are a Si12= StS =85 MW +5170 Mvar Stas = Sis +S = 5.0 MW 4515.0 Mvar Six = Sn +Ss2=08 MW +j1.60 Mvar 1 xamole 6.8 (smpedznees in pu on 120-MVA base The power flow diagram 1s shown in Figure 6.11, where real power direction 1s indicated by ~+ and the reactive power direction 1s indicated by -» The values within parentiteses are tne real and reactive losses inthe line. impedances converted 1 at 6 — 32, The load and ge + 1995 mi? oor oro 66.4 (0.8) 65.6. 189 Ts) ie 6) pt 02 105.0 90.0 44.8 43.2 3 + (15 ~ 192)(1.04 + 30) FIGURE 6.11 ene Power flow diagram of Example 6.7 (powers i9 MW and Muar) Example 6.8 Figure 6.12 snows the one-tine dingram of a simple three-bus power ssenerators at buses | and 3. The magnitude of voltage at bus 1 1 ad Pu. Voltage magnitude at bus 3 is fixed at 1.04 pu with a real power gener of 200 MW. A toad consisting of 400 MW and 250 Mvar is taken from bu: Line impedances are marked in per unit on a 100 MVA base, and the line cher susceptances are neglected. Obtain the power flow solution by the Gauss-Se ‘mothe including line Rows and line losses. The value of Q{! is usea as Qf" for the computation of voltage at bus 3. The complex vottage at bus 3, denoted by V2 is ealutates BOE sya toms” pee eee eee ee at i _ Basalt + (10 — 730)(1.05 + 50) + (26 — 792)(0.97462 ~ 7.042307) (2 — 362) = 1.03783 ~ j0.005170 Since | Val 18 held constant at 1.04 pu, onty the tmagmary part of ¥, ne, ff = 0.005170, and its real partis ot tual + nl vie ea ee mite (a6 ~ 783) = 0.971057 — jo.013132 =-31yy (ois + vas) — visi ~ va V}} = ~9{(1.039987 + 0.005170) [(1.039987 — j0.005170)(26 — 62) - (10 ~ 430)(1.05 + 0) ~ (16 — 732)(0.971057 — 0.04343: = 1.38796 + usVi + yes! i+ 7 + (10 — j30){1.05) + (16 ~ 532}(.971057 (26 - 762) ~ 40043432) = 1.03908 - 79.0730 {ETLOWS ANDLOSSES 219 setained, 5x 0 ' = 1.03996 — 0.00833 1996 — 0.00873 995 ~ j0.00893 1995 -~- j0.00900 996 ~ 40.0903 ex = $99 + j16.79 = 0.18 $70.58 220 6 POWERFLOW ANALYSIS (8.393) 170.968, (6.787) =f 101.947 [400 | 179.382 218.423, 2883 18 a 18.734 30.061 38.878 isb.asa 228.878 ogy) 229.082 19.693)! 250 148.053 Se (133) pin 8) 22.18 21.569 | 167-746 3 200) 146.177 FIGURE 6.13, Power fow diogram of Example 68 (powers in MW and Mua) 6.7 TAP CHANGING TRANSFORMERS Getermined by the angle difference ofthe terms deteomined mainly by the magnitude difference of led by use of tap ch Ina tap changing tansformer, transformer is represented by a series admuttance yj ratio, the per unit admuttance 1s different from botn sides of the tr adanittance must be modified to include the effect of tne off-nominal a transformer with admittance yin senes with an ideat transformer rep. the off-nominat tap rato I:a as-showa m Figure 8.14. pe 1s the ad Consider allowing for small adjustment in vottage of usually +10 percent, In the c: phase shifting transformers, a 18 a complex number. Consider a fit : power ‘on either side ofthe ideal transformer isthe same, ct follows i voltage goes through a positive phase angle shift, ine current will go tarough a negate phase angie shist, Thus, for the assumed direction of currents, we have y= ty k= (643) 16.44 ‘The current Ji 1s given by f= ylVi~¥e) s TAP CHANGING TRANSFORMERS 221 a Woy Hey e aa rt oa ea icone twee iba ign YQ wa i (6.46) an re 5.15 represents the ad- corresponds ‘0 the non-tap the transtormer. 222 6 POWERFLOW ANALYSIS: 6.8 POWER FLOW PROGRAMS Several computer programs have been developed for tne power flow solution of ‘Practical systems, Each method of solution consists of four programs. The pro- 4am for the Gauss-Seidel method is Ifgauss, which is prévéded by Ifybus, and is followed by busont and lineflow. Programs lfybus, busout, and lineflow are de- stgned to be used with «wo more power flow programs. These are linewton for the Newton-Raphson method and decouple for the fast decoupled method. The following isa bref desenption of the programs used in the Gauss-Seidei method, Mybus This program requires the line and transformer parameters and transformer ‘ap settings specified ia tne inpat file named linedata, It converts impedances ‘0 admtances and obtains the bus admittance matrix. The program is de- signed to handle parallel Ugauss ‘This program obtains the power flow sé and requires the files named busdata and linedata. Its de rect use of load and generation in MW and Mvar, bus voll and angic i degrees. Loads and generation are converted to per tes. the base MVA selected, A p power ofthe voltage-cot violation of reactive power either too high oF t20 Low. After a few tration method), te var calculated atthe generator buses are examined, reached, the voitage magnitude 1s adjusted in steps of 0.8 percent up percent to bang the var demand within the soscfied limits busout This program produces the bus output result ina taputated form, The bos output result includes the voliage magnitude and angle, real and te: ower of generators and loacs, and the shunt capacitor/reactor Mvar. Tot generation and total load are also included as outlined in tne sample ease. linellow This program prepares the line cutput data It is éesignec active and reactive power flow ensering the well asthe net power at each bus, Also included are th Josses mn the system, The output of this portion is also shown in the sample case 6.9 DATA PREPARATION jel method in the MAT- sd: power sysiem base yeximurn number of ite are basemya, follows: a power flow analysis by the ¢ following vanables m In onder to perfor TAB environs accuracy, accel, and manater, respectvely. Typ \ o in vaSenva = 100; accuracy = 0.001 = 80; secel = 1.5; song of each bus. Buses buses need not be entered in sequence. In quired. BUS DATA FILE ~ busdata The for 59.0ATA PREPARATION 225 224 4, POWER FLOW ANALYSIS MVA base, The last column is for the transformer tap setting; for lines, 1 must be entered in this column, The Hines may be entered in any sequence of onder with the only restriction being that if the entry isa transformer, te left bus number is assumed to be the tap side of the transformer, ‘The TEEE 30 bus system is used to demonstrate the data preparation and the use of the power flow programs by the Gauss-Seidel methd. Winding Transformer Example 6.9 '$ part of the Amencan Electric ing made available Regulated Bus Data BUS Voltage Min. Mvar Max. Mvar No. Magnituae Capacity Capacity 2 3 ~40 30 5 0 40 40 8 0 =10 40 u 2 6 24 3 1 6 24 ‘Transformer tap setting arc given in the table below. The left bus number 15 as- ‘soraed to be the tap side of the transformer. ‘Transformer Data “Transformer Tap Sewing Designation pu ‘The data for the mjectes ( cue to shunt capacitors 1s Injected Q due to Capacitors | Bus No, ‘Mvar 10 1S 24 43 226 5, POWER FLOW ANALYSIS Generation and toads are as given in the data prepared for use in the Mi environment in the matnx defined as busdata, Code 0, code 1, and code 2 for the load buses, the slack bus and the voltage-controlled buses, resp Values for basemva, accuracy, accel and maxiter must be species Line control commands required tna ‘may be used to save the output to the specified file name. The power flow data and the commands required are as follows. clear X clears all variables from workspace basenva = 100; accuracy = 0.001; accel = 1.8: maxater = 100 TEE 30-BUS TEST SYSTEM (Amerada Electric Power) ~ % Bus Bus Voltage Angle ~-Load-— ——~ Generator---Iayectea, 4 Wo code Nag. Degres Mi iivar! “it Hvar Qmin Qaax Yar busaatas(i i ide 0 6.0 0.9 Oo 1.043. 0 06 ot 0 ° ot ° 0 082 ° = Lane Data Bus 1 t Tenogatae tt 228 6. POWERFLOW ANALYSIS % Lfyous 4% Forms the ous admittance mateax Lf gauss % Power flow solutaon by Gauss-Seidel metaca, busout 4 Prints the power flow solution on tue screen Jineflow % Computes and displays the line flow and Losses ‘The Mgauss, busout and the linflow produce the following tabulated re: Power Flow Solution by Gauss-Seidel Method Naxanum Pover mismatca = 0,000951884 Wo. of iterations = 34 Bus Voltage angle 9 ----- Load-----__~-Generatacn-~ Tnyectea No. Mag. Degree MW Myar Mw Mvar var 1 1,060 0.000 0.000 0.000 260.950 -17.010 9.00 2 1.043 5.496 21.700 12.700 40.000 48.826 0.00 3 1.022 -8.002 2.400 1.200 0,000 0.900 0.00 4 1.013 9.659 7.600 1.600 0.090 0.000 0.00 5 1.010 -14.380 94.200 19.000 0.000 35.995 0.00 6 1.012 -11.396 0.000 0.000 0.000.000 9.00 7 1,003 -13.149 22.800 10.900 0.000 0,000 0,00 B 1.010 12.114 30.000 30.000 0.000 30,759 0.00 9 1.051 -14.432 0.000 9.000 0.080 0.000 0.00 10 1.044 16.024 5.800 2,000 0.900 9.000 19,00 41 1.082 ~14.432 0.000 0-000 9.000 16.113 9.00 42 1.087 -15.301 11.200 7.800 0.000 0.000 0.00 13 L.071 15.300 0.000 0.000 0.000 10.406 0.00 14 1.043 -16.190 6.200 1.600 0.000 0.000 9.00 AS 1.038 -16.276 8.200 2.500 9.000 9.000 9.00 16 1.045 -15.879 3.500 1.800 9.000 0.0c0 9.00 17 4.039 -16.187 9.000 6.800 0.009 0.000 0.00 18 1,028 -16.881 3.200 0.900 0,000 0.000 0.00 19 1,028 -i7.049 9.500 3.400 0.000 0.000 0,00 20 1.029 -16.85i 2.200 0.700 0,000 0.000 0.00 24 1.0382 -16.468 17.500 11.200 0,000 0.000 0.00 22 1.033 -16.455 9.000 0.000 9.000 0.000 9.00 23 1.027 -16.660 3.200 1.600 0,000 0.000 9.00 24 1.022 -16.829 8.700 6.700 0.000 9,000 4.30 25 1.019 -16.423 0.000 0.000 9.000 0.000 0.00 26 4.001 -16.835 3.500 2.300 0.009 9.000 0.00 27 1.026 -15.913 9.000 0.000 9.000 9.000 0.00 28 1.011 -12.056 9.000 0.000 0.000 9.000 9.00 29 1.006 -17.133 2.400 0.900 6.000 0,000. 0.00 30 0.994 -18.016 10.600 1.900 6.000 0.000 0.00 Total 283.400 126.200 200.950 125.099 23.30 Lane Flow and to crlaue-> Power at bus @ line flow --cane los-- Transformer from to MW Mvaz HVA MW Hvar tap i 260.950 -i7.010 261.504 2 477.743 -22.440 i7S.tzT 548i 10.887 3 89.197 5.125 83.351 2 ecT 7.078 2 18.300 1 -172.282 4 48.702 5 82.980 6 3 0.932 0.978 0.989 230. 6. POWERTLOW ANALYSIS 6 -37.170 1.317 37.193 0.368 -0.598 i ae 0.037 8 -30.000 0.759 30.010 6 -29.431 3.154 29.699 0,103 -0. 858 28 -0.570 -2.366 2.433 0.000 -4.368 i 0.079 9.010 9 9.000 9.000 0.000 6 27.687 8.911 29.086 0,000 1,593 14 0.003 -15.653 15.653 -0.000 0.461 9 aie 10 27,731 6.747 28.840 0.000 0.811 0034 40 *5.800 17.000 17.962 6 -15.828 0.623 15.840 0.000 1.279 20 9 -27.731 -5.936 28.359 9.000 0.811 20 9.018 3.569 9.698 © 0.081 0.180 17 5.397 4.393 6.920 0.014 0.037 24 18.723 9.046 18.551 0.1i0 90.238 a aad 227.882 4.487 8.811 0.0520. 107 0,001 i 0.000 18.113 16.113 9 0.003 16.114 16.114 -0.000 9.461 2 2 711.200 -7.600 13.479 4 44.131 -9.94: 45.237 0.000 4.636 13 -0.021 -10.274 10.274 0.000 0.132, 147.852 2.428 «8.219 0.07185, 23 0.083 1S 17.852 6.968 19.164 0.217 0.429, 0.012 16 7.208 3.370 7.985 0.053 0.112 13 0.000 10.406 10. 406 a 12 9.02 10.406 10.408 0.000 0.132 a4 8.200 4.600 6,403 120 -7.778 -2.273 4.103 0.074 0.158, i 15 4.592 0.708 1.742 0.006 0.006 7 oot 0,088 15 8.200 -2.500 8.873 0.049 42 17.634 -6.540 18.808 0.217 0.428 14 1.886 -0.702 1.734 0.006 0.008 = 18 6.009 1.741 6.256 0.039 0.079 26 9.086 23° 5.004 2.963 8.815 0.031 0.068 18 73,500 -1.800 3.936 ae 9.048 12 -7.152 -3.257 7.889 0.053 0.112 17 3.658 1.440 «3.931 0.012 0.027, 30 7.0931. 28 0.000 o. MW 18.192 5. 2 0.870 -2 6 18.720 -3. 29 -2.400 9, 27 6.093 -1 303.716. 30 10.600 -1 27 6.992 =i 73.683 -0, 29 Total less 152 675 663, 000 463, 003 519 900 513 601 900 359) 537 13 19 10. 7 3 +660 401 +286 000 984 082 106 863 278 764 769 06a 722 -0.000 0.086 0.162 0.000 9.000 9.060 0.086 0.034 0.162 0.034 47.594 Lato 0.162 0.304 1.310 4.368 13.085 0.162 0.063 0.304 0.063 22.283 6.10 NEWTON-RAPHSON POWER FLOW SOLUTION Because ofits quadrauc convergence, 0 the Gauss-Seidel method and 1s less problems. For large Jo the above equation, j includes bus 1 have ‘The complex power at bus fis P= 5Qs wh Newton's method is mathem: prone to divergence wi 1045 + 6 0.968 (6.48) Expressing this equation i polar form, we 649) 46.50) rR Substituting from (6.49) for Zan 6. { a Je Ja ds | aiv 234 5, POWERFLOW ANALYSIS snd the corresponding columns ofthe Jacobian matrix are eliminated. Accordingly, there are n — 1 real power constraints and n ~ 1 ~ m reactive power const ais, and the Jacobian matnx 1s of order (2n —2—m) x (2n 2 =m). Jy 1s of the order (n ~ 1) x (n= 1), Ja is ofthe order (n — 1) x (n 1 —m), Jp is of the order (n— 1m} x (71—1), and J is of the order (n— 1m} x (n= 1 my ‘The diagonal and the off-diagonal elements of Jy are foosdu + 714 cos(fis~ b+ ds) Ae (58) c08(045 — 5 + 54} (6.59) Leos ~ 55 +6)) yo (6.60) ‘The diagonal and the off-diagonal elements of Jy are sin dy — SO 1VjIIFislsin(Ay ~ 6; +0. i 1661) lly 6 +53) ake (6.2) The terms AP!” and AQ? are the difference between the schedied and calcu late valves, knows 25 he power residual, gen by APM = per — pi) 6.63) =a (on (6.65) (6.66) ‘The proceause for power flow sc: follows: ‘a-Raptison method is as 1, For toad buses, woere P' phase angles are set equal yge magnitudes and 14 00,12, | O4 = from (6.52) and (6.53) and oa) For voltage-conte (6.63), respectively from (6.52) and sare calculated us equation (6.54) 1s jon and Gaussta mally of- guar factor The new voltage magnitudes and phase an: (5.665. {rom (6.65) and 7 The process the spe on is demonstrated in Example 6.10 Obiain the power dow sol Exampie 6.3 y the Newtor-Raphsoa method 1 1e system of 236. 5, POWERFLOW ANALYSIS. Line impeaances converted and yas = 16 ~ 482. This results in the bus admittance matrix Your = | -104520 26-752 -16 + 792 20-750 -10+j20 10+ 530 104530 —16+ 792 26-362 Converting the bus admittance matrix to polar form with angles in radian yrelds 53.85465/-1,9029 22.36068/2.0344 31.6227821.8925 Yius = | 22.3606822.0344, 58.13777¢-1.1071 _38.7770922.0344 31.6227841.8925 35.77709/2.0344 67.23095¢—-1.1737 Fros reactive power at bus 2 are Pal ) Ppa [Yen] os(Ba1 ~ 5s +63) + [Vall Ya by +82) + VP 08 B33 =e i — Bs + 51) — [VE Yan| sim Ban ~ IVall¥al Vial sin( 65 ~ 82+ &3) fete Elements of the Jacobian matnx are obtained by taking paral denvatives of the above equations with respect to ba, dy and IV ar by = |Vall¥a [¥au| sin(Gas — 5a + 51) + Ja sin(@a ~ 82 + Ba} | Vail¥aahsim( ag — b+ 53) Py cas( 921 — 6 + 51) +2) a) cos( Boa ~ ba + ds) [Vall ¥o||¥oel s1n(O32 — 43 + 42) = |Va|l¥sa| cos (22 ~ 63 + b2) tances are yin = 10 ~ 720, mig = 10 ~ 730, (6.52) ana (6.53), the expressions for reat power at bus 2 and 3 anki the E S10. NEWION-R, cos(@ as ~ 82 + 4 ¥es| 6082p — 22 tof = =0.001795 0.000985, = -0.001767 For te third iteration, we have 62) = ~0.045263 + (0.001795) = ~o.04708 —0.007718 + (—0.000985) = —0.00870 IWP| = 0.973451 + (~0.001767} = og7i634 =0.000216, 51.596701 —31.693866 — 21.147447 0.000038 | =| —52.939865 65.597585 —15.351628 0.000143 —28.548205 17.396832 —47.954870 and 5p? = ~0,000038 iP) = -0.047088 + (—0.0000038) = -0 6,” = -0.0000024 6) = ~0.008708 + {—0.0000024) = 0.00: ay? = ~9.0000044 ‘The sorution converges i 3 iterations with a maximum power mismatch o 10° with Va = 0.971682 ~2.606° and ¥y = 1.04/—-0.4983" From (6.52) and (6.33), the expressions for teactive power at bus 3 and the slack bus ceal and reac. tive powers are = A Jcos Ais + ||| cos(Bia ~ 61 + 9) +|V [is] cos(Ais ~ 5: + 55} [Yial sin(O2 — 5; + Upon subsucution, we have Qs = 14617 pe P= 2.1882 pu Qi = 1.4085 pu the line flows are calcuiated in the same mannet as the line flow eater Im the Gauss-Seidel method descrited in Example 6.7, and the powe, liagram is as snown in Figure 6.13, A program named Ifhewton is ceveloped for power flow solution by the Newton-Raphson method for practical power systems. This program must be pre ceded by the Mybus program. busout and tineflow programs can be used to pont the load flow sotuton and the line flow results. The format is the same as te Gatiss-Seidel. The following is a bref Gesenption of the Ifnewton prograin 6.10. NEWEON-2.4P POR PLOW SOLUTION 239 Unewton This program method and requires Obtain ane Raphson me The data required 3s the samme as in Bx: swing commands e flow and Losses Bus Voltage angle Mette Hretetace, ayactea ag, Degree Hvar {tego 9.000 9.000 7.021 0.60 3 teas -s.ag7 21-700 3.822 0.00 1.922 -8.90a 2.400 9.309 9.000 0.00 240° 6, POWERFLOW ANALYSIS - 4 1.013 -9.661 7.600 1.600 0.000 0.000 0.00 5 1.010 ~14.381 94.200 19.000 0.000 38.975 0.00 6 1.012 -11.398 0.000 0.000 0.000 9.000 0.00 7 1,003 ~43.150 22.800 10.900 0.000 0.000 0.00 8 1.010 12.115 30.000 30.000 0.000 30.826 0.00 9 1.051 -14.484 0,000 0.000 0.000 0.000 0.00 10 1.044 16.024 5.800 2.000 0.000 9.000 19.00 42 1,082 -14.484 0.000 0.000 0.000 16.119 0.00 12 1.087 15.302 11.200 7.500 0.000 0.000 9.00 13 1.071 -15.302 0.000 0,000 0.000 10.423 0.00 14 1.042 -16.191 6.200 1.600 0.000 0.000 0.00 1S 1.038 -16.278 8.200 2.500 0.000 0.000 0.00, 16 1.045 -15.880 3.500 1.800 9.000 0.000 0.00 7 188 9.000 6.800 0.000 9.000 9.00 16 884 3.200 0.900 9.000 0.000 9.00 19 052 9.500 3.400 0.000 0.000 9.00 20 852 2.200 0.700 0.000 © 0.000 9.00 2 468 17.500 11.200 0.000 0.000 0.00 22 455 0.000 0.000 9.000 0.000 9.00 23 662 3.200 1.600 9.009 0.000 9.00 24 830 8.700 6.700 9.000 9.000 4.30 28 424 0.000 9.000 0.000 9.000 0.00 26 842 3.600 2.300 0.000 0.000 0.00 2 912 0.000 0.000 9.000 0.000 9.00 28 087 9.000 9.900 9.000 + 0.000 0.00 238 136 2.400 0.900 0.000 0.000 9.00, 30 01S 10.600 1.900 9.000 0.000 9.00 Total 283.400 126.200 300.998 125.194 23.20 ‘The output of the lineffow is the same as the the power mismatch as dictated by the Ne 1 flow output of Example 6.9 with Raphson method. 6.11 EAST DECOUPLED POWER FLOW SOLUTION Power system transmission lines have a very high X/R rauo, Fer such 2 system, real power changes AP are less sensitive to changes mn the voltage magnitude and are most sensitive to changes in phase angle Ad, Similary, reactive power 18 less sensitive to changes in angle and are mainly dependent on changes in voitags ‘magnitude. Therefor, it is reasonable to set elements Ja and Jy of the Jacobian ‘matnix to zer0, Thus, (6.54) becomes AP)]_[A 0]f a6 aQ}=lo | Lavy (6.68) In a typical power s¥ Q. Fanner samplif (6.69) (6.20) 6.7) Similar, the diagona elements of Jg described by (6.61) may be watien as in(Bij — 54 +65) ing the second term of the above equation with —Qy, as given by (6.53), (67 Likewise in (6.62), assuming Ou; ~ 6, +55 = yields oa = (618) avi = With these assumptions, equations (6.69) ana (6,70) take the following forma AP pt as (575) Ag pr aly) (7091 ivi Here, B’ and B” are the emaginary part of the bus admittance matrix Ving the elements of this matnx are constant voltage-controlled buses where |Vi| and P, ar the corresponding row and cotumn of Yous 8 sof order of (n 1 ~m), where mis tne number of voltage-regulated buses. Therefore, 1n the fast decoupied power flow algorithm, the successive voltage magnitude and phase angie changes are AlV| = -18' (6.78) Iv +The fast cecoupied power flow solution requires more Wterations than the Ne Raphson method, but requires considerably Jess time peri flow solution is obtained very rapidly. This techmaque is very use analysis where numerous cutages are to pe simulated or a power flow requited for of east /ER FLOW SOLUTION 243 Example 6.12 Obtain the power flaw s« ample 6.8, 10d for the system of Ex. Exampte 6.10 is 0 2 | Ws susceplance matrix bus 2 ang 2 and the a ‘2AM 6, POWERFLOW ANALYSIS ‘The fast decoupled power flow algorithm given by (6.77) becomes Asp] __ {0.028182 —0.014545 ae a ree aa? ~~ | 0.014545 ~0.023636 || + * | -0.008909 Tot Since bus 31s a regulated bus, the corresponding row and column of BY are elimi nated and we get From (6.78), wenave a1) [a2 Alval =- (FI =F = -0,0042308 ‘The new bus voltages inthe fist teranon are = 0+ (0.060489) = -0.060483 age phase angles are in radians. The process 1s continued unt uais are within a specified accuracy, The resutt is tabulated in the table BP; AP; “ZR60000 1438800 0.175895 -0,070951 0.640309 “GD6DAES “0.08909 0.056496 0.007952 -0.044194 -0.008690 > 0.044802 -0.008986 0.021395 0.008713 0.153368 0.008645, 6.000520 0.008702 0.035980 -0,008720 0.971732 0.000948 -0.008707 0.971762 -9.008432 0.008702 -0.000470 0,000510 0.008705 0.00 0.047084 .0.008706 0.000163 -0.047063 -0.008706 0.047068 0.000330 0.008706 0.971680 _-0.000053 0.000048, o.009;51 0.000250 Converting phase angles to degrees the final solution is V3 = 0.97168¢ —2.696° and Vq = 1.042 0.4988". Using (6.52) and (6.53) as in Example 6.10, wie reactive is tne power flow 613 for the IEEE um the power flow si Pled metho Data requized is the same as sn Example 9

You might also like